American Board of Family Medicine

IN-TRAINING EXAMINATION

TIME–4 HOURS

Publication or reproduction in whole or in part is strictly prohibited. Copyright © 2013 The American Board of Family Medicine. All rights reserved. 1. A 52-year-old female with a history of hypertension and hypercholesterolemia presents with mild , weakness, and body aches. Her only medications are atorvastatin (Lipitor) and chlorthalidone. Her previously normal serum creatinine level is now 2.6 mg/dL (N 0.64–1.27). Her BUN level is 32 mg/dL (N 6–20) and her serum is clear without pigmentation. The urine dipstick is positive for , but a microscopic examination is negative for WBCs, RBCs, and casts.

The most likely diagnosis is

A) allergic interstitial nephritis B) glomerulonephritis C) hemolysis D) pyelonephritis E) rhabdomyolysis

2. A 36-year-old male with a history of ankylosing spondylitis and atrial fibrillation presents with a 3-week history of cough with hemoptysis, anorexia, night sweats, and an 11-lb weight loss. On examination he has rales in the right upper lobe, but there is no lymphadenopathy or hepatosplenomegaly. A chest radiograph shows a cavitary lesion in the right lung apex, with mediastinal hilar lymphadenopathy. His chronic disease symptoms have been well controlled with a combination of meloxicam (Mobic), adalimumab (Humira), esomeprazole (Nexium), ondansetron (Zofran), docusate sodium (Colace), and amiodarone (Cordarone).

Which one of the patient’s medications is most likely contributing to his current problem?

A) Adalimumab B) Amiodarone C) Esomeprazole D) Meloxicam E) Ondansetron

3. You have just finished giving a prescription with instructions to a 28-year-old male from El Salvador who speaks limited English. You gave the instructions with the aid of an interpreter, but are concerned that the patient might not fully understand them.

Which one of the following is the best course of action?

A) Refer the patient to a website about his condition B) Repeat the instructions slowly to the interpreter and ask him or her to speak clearly to the patient C) Contact a family member who speaks English and ask him or her to repeat the instructions to the patient D) Ask the patient to repeat the instructions to you in his own words

1 4. Which one of the following accurately describes the classic rash of erythema migrans?

A) Scattered individual purple macules on the ankles and wrists B) An annular rash with a bright red outer border and partial central clearing C) A dry, scaling, dark red rash in the groin, with an active border and central clearing D) A diffuse eruption with clear vesicles surrounded by reddish macules E) A migratory pruritic, erythematous, papular eruption

5. A patient with chronic atrial fibrillation treated with dabigatran (Pradaxa) sees you for follow-up. She says she can no longer afford the dabigatran and would like to switch to warfarin (Coumadin). She has normal renal function.

Which one of the following would be the most appropriate approach?

A) Start warfarin and stop dabigatran when her INR is 2.0–3.0 B) Start warfarin now and stop dabigatran in 3 days C) Stop dabigatran, start warfarin, and start low molecular weight heparin and enoxaparin (Lovenox) every 12 hr until her INR is 2.0–3.0 D) Stop dabigatran for 24 hr and then start warfarin E) Hospitalize the patient, stop dabigatran, start warfarin, and treat with heparin until her INR is 2.0–3.0

6. A 27-year-old male requests your advice regarding colon cancer screening. His brother died of colon cancer, which was diagnosed at the age of 40.

You suggest that he begin colonoscopy screening

A) now B) at age 30 C) at age 40 D) at age 45 E) at age 50

7. Which one of the following is the major mechanism of action of metformin (Glucophage)?

A) Stimulation of pancreatic insulin release B) Inhibition of glucose production by the liver C) Inhibition of carbohydrate absorption in the small intestine D) Improved insulin sensitivity of skeletal muscle

8. Which one of the following medications is most appropriate for treating moderate to severe shortness of breath in a hospice patient with lung cancer?

A) Dexamethasone B) Haloperidol C) Scopolamine D) Morphine

2 9. A 35-year-old nulligravida sees you for preconception counseling. She has hypothyroidism treated with levothyroxine (Synthroid), and her most recent TSH level was in the therapeutic range. She has no symptoms of hypothyroidism.

Which one of the following is the patient most likely to require if she becomes pregnant?

A) A decreased dosage of levothyroxine B) An increased dosage of levothyroxine C) The addition of liothyronine (Cytomel) D) Substitution of desiccated thyroid hormone preparation (Armour Thyroid) for the levothyroxine

10. Which one of the following is considered to be the highest strength of recommendation by the Strength of Recommendation Taxonomy (SORT) used by family medicine journals?

A) Expert opinion B) A consensus guideline C) A retrospective cohort study D) Multiple good quality randomized, controlled trials

11. Which one of the following tumors is most likely to cause hormonally induced hypercalcemia?

A) Squamous cell carcinoma of the lung B) Pheochromocytoma C) Medullary thyroid cancer D) Prostatic carcinoma

12. A 32-year-old meat cutter comes to your office with persistent symptoms of nausea, vomiting, and diarrhea, which began about 36 hours ago on the last day of a 5-day Caribbean cruise. His wife was sick during the first 2 days of the cruise with similar symptoms. On the ship they both ate the “usual foods” in addition to oysters. Findings on examination are negative, and a stool specimen is negative for white blood cells.

Which one of the following is the most likely cause of his illness?

A) Escherichia coli B) Rotavirus C) Norovirus D) Hepatitis A E) Giardia

3 13. A 47-year-old postmenopausal female falls while carrying groceries into her house and sustains a right distal radial fracture. A chemistry panel reveals a calcium level of 11.2 mg/dL (N 8.6–10.6) and further evaluation leads to a diagnosis of primary hyperparathyroidism.

Which one of the following is the best course of treatment for this patient?

A) Estrogen replacement therapy B) Long-term bisphosphonate therapy C) Daily furosemide treatment with increased oral fluids D) Elimination of calcium and vitamin D from the diet E) Referral to a surgeon for consideration of parathyroidectomy

14. Activated protein C resistance (factor V Leiden) is most commonly found in patients with

A) hemolytic anemia B) carcinoma of the lung C) familial hypercholesterolemia D) venous thrombotic disease E) cystic fibrosis

15. Which one of the following is most consistent with a diagnosis of asthma?

A) Reduced FEV1 and a decreased FEV1/FVC ratio B) Reduced FEV1 and a normal FEV1/FVC ratio C) Reduced FEV1 and an increased FEV1/FVC ratio D) Reduced FVC and a normal FEV1/FVC ratio E) Reduced FVC and an increased FEV1/FVC ratio

16. A 43-year-old female presents to your office 2 days after discovering a rash on her back, shown below. Which one of the following treatments will decrease her chances of developing long-term sequelae?

A) Amitriptyline B) Gabapentin (Neurontin) C) Oral corticosteroids D) Topical corticosteroids E) Oral acyclovir (Zovirax)

4 17. You receive a telephone call from the mother of a 5-year-old female. The child has had diarrhea and a decreased appetite for the past 2 days. She is still playing some. The mother reports no vomiting, but says her daughter has complained of a dry mouth and does not have tears when she cries. You suspect that the child may be mildly dehydrated.

Which one of the following would you advise?

A) Increased water intake B) Clear liquids with sodium, such as chicken broth C) An over-the-counter oral rehydration solution D) Intravenous fluids in the emergency department E) Loperamide (Imodium)

18. A 42-year-old male with a history of chronic hepatitis C develops left leg cellulitis and is treated with cephalexin (Keflex). He returns to your office 5 days later for follow-up, and the cellulitis is responding favorably to treatment. However, the patient has a generalized maculopapular rash and a low-grade fever, which he says began 3 days ago. He also complains of arthralgias. You admit him to the hospital for further evaluation.

His serum creatinine level is 3.2 mg/dL (N 0.6–1.5), which is elevated from his baseline level of 0.8 mg/dL. A urinalysis is normal, except for the presence of occasional eosinophils. The remainder of his evaluation, including liver enzyme levels and renal ultrasonography, is normal.

Which one of the following is the most appropriate next step in the management of this patient?

A) A postvoid residual urine volume B) A hepatitis C viral load and genotype C) Discontinuing cephalexin D) Antibiotics to cover methicillin-resistant Staphylococcus aureus (MRSA) E) Aggressive fluid resuscitation with normal saline

19. Which one of the following medications is most likely to cause hypokalemia?

A) Albuterol (Proventil, Ventolin) B) Doxazosin (Cardura) C) Erythromycin D) Felodipine (Plendil) E) Lisinopril (Prinivil, Zestril)

5 20. An abandoned infant is brought to the hospital for evaluation. Based on the presence of a dried remnant and her overall appearance, you believe her to be no more than 5 days of age. A thorough examination is normal except for a finding of bilateral conjunctival erythema and exudate. A Gram stain of the exudate is remarkable for numerous WBCs, very few of which are noted to contain gram-negative diplococci.

Which one of the following treatment options is most appropriate?

A) Application of moist, warm saline eye compresses B) Irrigation of both eyes with povidone-iodine (Betadine) C) One-time application of ophthalmic erythromycin ointment into both eyes D) Instillation of silver nitrate solution into both eyes E) Intramuscular injection of ceftriaxone (Rocephin)

21. Which one of the following has been shown to have a beneficial effect for symptoms of the common cold in an adult?

A) Diphenhydramine (Benadryl) B) Ipratropium (Atrovent) nasal spray C) Intranasal zinc D) Intranasal corticosteroids E) Systemic corticosteroids

22. A 50-year-old white male is injured while mountain biking. The CT scan of the patient’s abdomen shown below is most consistent with which one of the following?

A) Rupture of the spleen B) Subcapsular hematoma of the spleen C) Subcapsular hematoma of the kidney D) Ruptured hollow viscus E) Herniated vertebral disc

23. A 21-year-old primigravida at 10 weeks gestation has a negative titer for rubella. The best procedure to follow is to

A) institute a (-globulin regimen and maintain it throughout her B) administer rubella vaccine after 12 weeks gestation C) administer rubella vaccine immediately post partum D) administer rubella vaccine 12 weeks post partum

6 24. A postmenopausal female who has recently been diagnosed with hypertension returns for follow-up 3 months after the initiation of therapeutic lifestyle changes. Her blood pressure has improved but remains higher than goal at 142/90 mm Hg, and pharmacologic treatment is indicated. The patient has a family history of osteoporosis.

Which one of the following may slow the demineralization of bone in this patient?

A) An ACE inhibitor B) An "-blocker C) A $-blocker D) A calcium channel blocker E) A thiazide diuretic

25. An otherwise healthy 1-year-old male is brought to your office because of increased respiratory effort, wheezing, and rhinorrhea. He has no fever. On examination he is found to have an increased respiratory rate and mild retractions. A chest film shows no foreign body or infiltrates. His O2 saturation is 94%.

Management should include which one of the following?

A) A trial of nebulized albuterol (AccuNeb) B) Nebulized epinephrine (Asthmanefrin) C) Oxygen D) Antibiotics E) Corticosteroids

26. An 80-year-old female is admitted to your service at a skilled nursing facility 5 days after repair of a hip fracture. When you review her records you note that she has not received any previous treatment for osteoporosis. You are considering ordering zoledronic acid (Reclast) to reduce her risk of another fracture.

Which one of the following should be evaluated before administering zoledronic acid to this patient?

A) Vitamin D levels B) Liver enzyme levels C) Estimated glomerular filtration rate D) A CBC

7 27. A 58-year-old male has a history of type 2 diabetes mellitus that is not well controlled. He has recently developed mild hypertension that has not been controlled by lifestyle changes. You prescribe lisinopril (Prinivil, Zestril), 20 mg daily, for the hypertension and 2 months later you note that his serum creatinine level has increased from 1.25 mg/dL to 1.5 mg/dL (N 0.64–1.27) and his blood pressure has decreased from 142/88 mm Hg to 128/78 mm Hg.

Which one of the following should you do now?

A) Continue the current dosage of lisinopril B) Decrease the dosage of lisinopril to 10 mg C) Increase the dosage of lisinopril to 40 mg D) Discontinue lisinopril and initiate chlorthalidone E) Discontinue lisinopril and initiate losartan (Cozaar)

28. A 68-year-old female presents with recent poor oral intake, fatigue, and confusion. Osmotic demyelination syndrome (central pontine myelinolysis) and permanent neurologic deficits could result from overly rapid correction of which one of the following abnormalities?

A) Hyperglycemia B) Hyperkalemia C) Hypokalemia D) Hypernatremia E) Hyponatremia

29. A 22-year-old primigravida presents for routine at 18 weeks gestation. She is frustrated because of increased pigmentation on her face consistent with (chloasma).

Which one of the following would you recommend for this patient?

A) Use of a high-potency broad-spectrum sunscreen B) Use of hydroquinone for 4 weeks C) Postpartum use of oral contraceptives D) Avoiding future use of topical retinoids E) Increased surveillance for skin cancer beginning at age 40

8 30. A 40-year-old white male was seen 4 weeks ago for a sudden onset of cough and shortness of breath. At that visit his O2 saturation was 92%, but his examination and a chest radiograph were normal. You prescribed azithromycin (Zithromax) and an albuterol inhaler (Proventil, Ventolin). Ten days later he was feeling well and his oxygen saturation was 97%. Today he returns to the office with a dry cough and shortness of breath.

On examination he has rare inspiratory rales that clear with deep breaths, and he has an O2 saturation of 86%. A chest film and a D-dimer test are normal. Pulmonary function tests show significant restriction that improves only minimally with albuterol. He has not been exposed to anyone with a similar illness, has no history of asthma, and has no smoking history or occupational exposure. However, he reports that 2 months ago his home was flooded after a heavy rain, and he has been tearing out carpeting that was ruined by the flood.

Which one of the following is the most likely diagnosis?

A) Persistent asthma with acute exacerbations B) Legionnaires’ disease C) Pulmonary D) Hypersensitivity pneumonitis

31. Which one of the following metabolic abnormalities is most likely to be seen in patients with stage 4 kidney disease?

A) Hyperaldosteronism B) Hyperparathyroidism C) Hypothyroidism D) Hypogonadism E) Type 2 diabetes mellitus

32. Which one of the following is the usual reservoir for hantavirus?

A) Prairie dogs B) Jackrabbits C) Deer mice D) Ground squirrels

33. The skin lesion shown below is characteristic of which condition often associated with a drug reaction?

A) Id reaction B) Erythema multiforme C) Discoid lupus D) Granuloma annulare E) Pyoderma gangrenosum

9 34. A new home urine test is designed to detect a particular type of cancer. The gold standard test for this cancer is a biopsy, but a biopsy is more costly than the urine test, is invasive, and is associated with a number of adverse side effects. To test the effectiveness of the home urine test, 104 people took the test and then agreed to a biopsy. When the study was concluded, 77 people tested negative and 27 tested positive on the urine test. Biopsies were positive in 18 individuals, 8 of whom tested negative on the urine test.

What is the negative predictive value of the home urine test, rounded to a whole number?

A) 20% B) 37% C) 56% D) 80% E) 90%

35. An 85-year-old male is admitted to a nursing home due to weakness, debility, and limitation of activities of daily living (ADLs) after being hospitalized for acute community-acquired pneumonia. He previously lived with his wife independently and his goal is to return home when he is strong enough. He has a history of coronary artery disease, type 2 diabetes mellitus controlled with diet, hypertension, and chronic diastolic heart failure, but he has no symptoms related to these chronic problems. His appetite is poor and he has lost a significant amount of weight. His admission diet order from the hospital was a cardiac diet.

Which one of the following would be the most appropriate diet for this patient?

A) A regular diet B) An American Heart Association diet C) A diet with no added salt D) An 1800-calorie/day American Diabetes Association diet E) A diet with no concentrated sweets

36. In women with polycystic ovary syndrome, the risk is increased the most for carcinoma of the

A) breast B) cervix C) colon D) endometrium E) ovary

37. Which one of the following cardiac rhythm abnormalities is most common in patients with anorexia nervosa?

A) Atrial fibrillation B) Ventricular fibrillation C) Sinus bradycardia D) Sinus tachycardia E) Paroxysmal supraventricular tachycardia

10 38. An asymptomatic 32-year-old male requests screening for sexually transmitted diseases. A nucleic acid amplification test is performed on a urine sample, and the results are positive for gonorrhea and negative for Chlamydia. The patient has no known drug allergies.

Which one of the following is the recommended treatment for this patient?

A) Ceftriaxone (Rocephin), 125 mg intramuscularly B) Ceftriaxone, 250 mg intramuscularly C) Ceftriaxone, 250 mg intramuscularly, plus azithromycin (Zithromax), 1 g orally D) Ceftriaxone, 125 mg intramuscularly, plus doxycycline, 100 mg orally twice daily for 7 days E) Ciprofloxacin (Cipro), 500 mg orally

39. Which one of the following vaccines is CONTRAINDICATED in immunocompromised adults?

A) Herpes zoster B) Human papillomavirus C) Meningococcal D) Pneumococcal polysaccharide E) Tdap

40. You are the attending physician at a long-term care facility. A new resident, an 85-year-old female, presents for an initial evaluation. Upon reviewing her history, you find that she is on 18 different medications. Until you can obtain additional history and medical records, you decide to stop or decrease some of her medications and monitor her response.

Which one of the following would be most appropriate to stop or decrease initially?

A) Sertraline (Zoloft), 25 mg daily B) Acetaminophen/diphenhydramine (Tylenol PM), 500 mg/25 mg daily C) Dipyridamole/aspirin (Aggrenox), 200 mg/25 mg daily D) Digoxin, 0.125 mg every other day E) Omeprazole (Prilosec), 20 mg daily

41. A 25-year-old primigravida asks about pain management during labor. You inform her that use of regional analgesia during labor

A) increases the likelihood of cesarean delivery B) increases the risk for instrument-assisted vaginal delivery C) provides less pain relief than opioid analgesia D) lowers 1-minute Apgar scores

11 42. A 28-year-old female visits your office because she is worried about episodic abdominal pain and altered bowel habits that have been present intermittently for the past year. She describes the character of her abdominal pain as “crampy,” and says it can become quite severe. Defecation brings some relief, but she sometimes must pass several loose, watery stools before the pain resolves. She occasionally notes mucus in her stool and sometimes feels that evacuation is incomplete even though she is unable to pass more stool. Her body weight is stable, and findings on the abdominal examination are completely normal.

Which one of the following diagnostic tests does current evidence support in this case?

A) Hydrogen breath testing B) Testing for celiac disease C) Stool testing for ova and parasites D) Thyroid function testing E) CT of the abdomen

43. Which one of the following conditions presents an unacceptable health risk for combined oral contraceptive use?

A) Migraine with aura B) Endometrial hyperplasia C) Breastfeeding 1–6 months post partum D) Chronic hepatitis C E) Previous laparoscopic banding weight-loss surgery

44. A 56-year-old male with diabetes mellitus, hypertension, and chronic renal insufficiency presents for follow-up of his chronic medical conditions. Results of his most recent metabolic panel included an estimated glomerular filtration rate of 30 mL/min/1.73 m2 (N >60) and a calcium level of 10.4 mg/dL (N 8.5–10.2). Medication reconciliation reveals he is not taking the sevelamer (Renagel, Renvela) prescribed by the consulting nephrologist.

You explain to the patient that he should be taking sevelamer to lower his serum calcium. The drug accomplishes this by

A) blocking the effect of parathyroid hormone B) blocking excessive vitamin D levels, thus decreasing intestinal calcium absorption and increasing renal calcium excretion C) blocking intestinal absorption of phosphate, which lowers parathyroid hormone secretion D) directly blocking excessive calcium absorption in the intestines E) directly increasing the renal excretion of both calcium and phosphate

12 45. A 16-year-old afebrile, otherwise healthy female presents with a 4-day history of right ear pain. She says she has spent a fair amount of time swimming recently. Traction on the pinna causes pain. The erythema and inflammation is limited to the ear canal but there is too much edema to easily visualize the tympanic membrane.

Which one of the following would be the most appropriate treatment?

A) Amoxicillin B) Amoxicillin/clavulanate (Augmentin) C) Amoxicillin/clavulanate plus acetic acid 2% topically D) Ciprofloxacin 0.3%/dexamethasone 0.1% (Ciprodex) topically

46. An obese 10-year-old male with tonsillar hypertrophy is brought to your office because of snoring. There is no history of recent or past visits for tonsillitis. Polysomnography shows moderate obstructive sleep apnea syndrome.

Which one of the following is the treatment of choice for this patient?

A) Continuous positive airway pressure (CPAP) B) Intranasal corticosteroids C) Extended antibiotic therapy D) Adenotonsillectomy

47. For which type of renal calculus is acidification of the urine indicated?

A) Cystine B) Uric acid C) Calcium oxalate D) Calcium phosphate

48. An 11-year-old male is brought to your office for evaluation of bilateral posterior heel pain that has occurred for the past few months. He plays basketball and soccer several times a week and the pain begins several minutes into each of these activities. There is no pain at rest or with walking. The patient has not noticed any numbness, tingling, or weakness.

On examination you find no swelling or tenderness of the heel or Achilles tendon. Reflexes, strength, and range of motion at the ankle are intact, but he does have bilateral posterior heel pain when you passively dorsiflex the ankles.

Which one of the following is the most likely diagnosis?

A) Achilles tendinopathy B) Calcaneal apophysitis C) Plantar fasciitis D) Heel pad syndrome E) Tarsal tunnel syndrome

13 49. A 75-year-old male comes to your office for a routine follow-up visit for hypertension. He is asymptomatic but your evaluation reveals atrial fibrillation. An echocardiogram is normal except for mild left ventricular hypertrophy.

Which one of the following would be most appropriate at this point?

A) No anticoagulation B) Medical cardioversion C) Anticoagulation with aspirin D) Anticoagulation with warfarin

50. A 63-year-old male presents for a preoperative evaluation prior to total knee arthroplasty. He underwent coronary artery angioplasty and stent placement 3 years ago following an episode of angina and an abnormal exercise stress test. His current medications include aspirin, atorvastatin (Lipitor), and lisinopril (Prinivil, Zestril). He runs 2 miles three times/week without difficulty. He has no history of heart failure, diabetes mellitus, kidney disease, or cerebrovascular disease. An EKG and preoperative blood tests, including kidney function tests, are normal.

Which one of the following is indicated prior to surgery to decrease this patient’s perioperative risk?

A) Performing noninvasive cardiac stress testing B) Continuing his statin therapy C) Discontinuing aspirin D) Starting a $-blocker E) Starting clopidogrel (Plavix)

51. A 14-year-old male has open and closed comedones without evidence of surrounding inflammation on his face and upper back. Which one of the following is the most appropriate initial treatment?

A) Topical antibiotics B) Topical retinoids C) Oral antibiotics D) Oral isotretinoin

14 52. A 13-year-old African-American male is brought to your office by his mother for a limp that she has noticed for about 1 week. The patient admits to vague right-sided hip and knee pain present only with activity. He says the pain has never awakened him from sleep and is never present at rest. Neither the mother nor the patient has noticed any systemic symptoms such as fever, night sweats, weight loss, or appetite changes. There is no recent history of trauma.

On examination the patient’s weight is in the 90th percentile and his height is in the 50th percentile. He has an antalgic gait. Examination of the left hip is normal. Examination of the right hip is significant for decreased internal rotation. The right hip externally rotates involuntarily with passive flexion. There is no external deformity and no skin changes are noted. He has a negative FABER test. Palpation of the bursa and bone does not elicit pain. Examination of both knees is normal. Plain radiographs confirm your clinical impression.

Which one of the following is the most appropriate next step in the management of this patient?

A) Reassurance with close follow-up B) Physical therapy C) Injection of the sacroiliac joint D) Hospital admission for intravenous antibiotics E) Surgery

53. A 57-year-old previously healthy menopausal female presents to your office with a 1-year history of palpitations and an unintentional 10-lb weight loss. A review of systems is negative for tremors or visual changes. Vital signs include a blood pressure of 129/85 mm Hg and a heart rate of 110 beats/min. A physical examination is otherwise unremarkable except for a nontender, diffusely enlarged thyroid with no distinct nodules, and mild proptosis.

: Laboratory studies are significant for a TSH level<0.01 U/mL (N 0.60–3.30), a free T3 level of 14.51 pg/mL (N 2.0–3.5), and a free T4 level of 4.52 ng/dL (N 0.71–1.40). A thyroid- stimulating immunoglobulin test is positive.

In addition to a $-blocker, which one of the following is the most appropriate initial management?

A) Radioactive iodine ablation B) Thyroidectomy C) Methimazole (Tapazole) D) Propylthiouracil (PTU)

15 54. A 39-year-old male with a BMI of 41 kg/m2 is interested in weight loss. His medical history includes adequately controlled type 2 diabetes mellitus, well-controlled hypertension, hyperlipidemia, and obstructive sleep apnea. He has no history of coronary artery disease or COPD.

Which one of the following is likely to be most effective for long-term weight loss in this patient?

A) A very low calorie diet B) Increased physical activity C) Frequent, long-term weight-loss counseling D) Pharmacotherapy E) Bariatric surgery

55. A 45-year-old female with no significant past medical history presents to your office with 2 weeks of worsening pain in her right arm. For the past 2 months she has worked on a plastics manufacturing assembly line. A physical examination reveals no swelling and a normal range of motion. She has normal strength in the upper extremity but she experiences increased pain with extension of her right wrist against resistance. Palpation reveals marked tenderness over the lateral epicondyle of the right arm.

Which one of the following is most likely to improve the patient’s long-term outcome?

A) Physical therapy B) Regular physical activity using her hands and arms C) Use of an inelastic, nonarticular proximal forearm strap D) Modifying her work routines

56. A 76-year-old white male with heart failure is admitted to the hospital for the third time in a year. He responds to treatment with intravenous furosemide (Lasix), oxygen, and morphine. When he is discharged, his medications include carvedilol (Coreg), 25 mg twice daily; furosemide, 40 mg daily; and lisinopril (Prinivil, Zestril), 40 mg daily. He is also placed on a low-salt diet.

Which one of the following is most likely to help prevent future admissions and decrease overall medical costs for this patient during the next year?

A) Case management by a heart failure specialist nurse B) Nursing home admission C) Adding amiodarone (Cordarone) D) Increasing the dosage of lisinopril

16 57. A 56-year-old African-American male with long-standing hypertension and a 30-pack-year smoking history has a 2-day history of dyspnea on exertion. A physical examination is unremarkable except for rare crackles at the bases of the lungs.

Which one of the following serologic tests would be most helpful for detecting left ventricular dysfunction?

A) B-type natriuretic peptide (BNP) B) Troponin T C) C-reactive protein (CRP) D) D-dimer E) Cardiac interleukin-2

58. A 35-year-old male presents with acute low back pain after he spent a weekend building a storage shed in his backyard. He has no neurologic symptoms, and the pain does not radiate into either leg.

Which one of the following has been shown to be a useful treatment in this situation?

A) Bed rest B) Acupuncture C) Lumbar traction D) Cyclobenzaprine (Flexeril) E) Methylprednisolone (Medrol)

59. For which one of the following respiratory infections should antibiotic therapy be initiated immediately upon diagnosis?

A) Bronchitis B) Epiglottitis C) Laryngitis D) Rhinosinusitis E) Tracheitis

60. A mother meets you in the emergency department with her 3-week-old infant. The infant was delivered at term, with an uneventful prenatal and postnatal course to this point. The mother reports that the infant stopped breathing for 20–25 seconds, and that his lips and tongue appeared bluish. There was no coughing, choking, or congestion, but the child seemed “limp.” The episode ended when the mother vigorously stimulated her child and he started crying. On examination, the child appears normal.

Which one of the following would be most appropriate at this point?

A) Reassurance and no further evaluation B) Discharge with a home apnea monitor C) Hospital admission for observation D) Mandatory referral to child protective services E) Direct laryngoscopy to rule out a foreign body

17 61. A 30-year-old female at 36 weeks gestation has a positive culture for group B Streptococcus. Her past medical history is significant for the development of a nonurticarial rash in response to penicillin.

Which one of the following is most appropriate for intrapartum antibiotic prophylaxis in this patient?

A) Azithromycin (Zithromax) B) Clindamycin (Cleocin) C) Vancomycin (Vancocin) D) Ampicillin E) Cefazolin

62. For the prevention of ischemic stroke in patients at low risk for gastrointestinal , the U.S. Preventive Services Task Force recommends aspirin for

A) men age 45–79 B) men age 55–79 C) women age 45–79 D) women age 55–79 E) no one, regardless of sex or age

63. Pretibial myxedema is a cutaneous manifestation of

A) subclinical diabetes mellitus B) collagen vascular disease C) hyperlipidemia, type III D) ischemia E) Graves disease

64. A 45-year-old male presents with a 3-month history of hoarseness. He denies any other complaints and has not been ill recently. He is not on any medication, has no history of chronic medical problems, and does not smoke cigarettes or drink alcohol.

Which one of the following would be the most appropriate management of this patient?

A) Voice rest for 1 month B) Laryngoscopy C) A trial of a proton pump inhibitor D) A trial of inhaled corticosteroids E) Oral corticosteroids

18 65. In which one of the following populations does the U.S. Preventive Services Task Force support ultrasound screening for abdominal aortic aneurysm?

A) All men age 55–75 B) Males age 55–75 who currently smoke C) Patients of both sexes age 55–75 who currently smoke D) Men age 65–75 who have ever smoked E) No population group

66. A previously healthy 74-year-old male presents to the emergency department with a fever and altered mental status. His illness began 2 days ago with symptoms of fever, malaise, body aches, reduced appetite, nausea, and diarrhea. His temperature is 39.6°C (103.3°F) in the emergency department and his examination is nonfocal. Initial laboratory studies include a sodium level of 131 mEq/L (N 135–145) and a WBC count of 14,200/mm3 (N 4500–11,000) with a neutrophilic predominance. Blood and urine cultures are obtained and he is admitted to the hospital for observation.

The next morning he develops a productive cough and shortness of breath. You order a chest radiograph, which shows patchy consolidation of the bilateral bases.

Which one of the following is the most likely cause of this patient’s condition?

A) Chlamydophila pneumoniae B) Legionella pneumophila C) Mycoplasma pneumoniae D) Streptococcus pneumoniae

67. A 62-year-old male underwent percutaneous coronary intervention and placement of two stents for a myocardial infarction yesterday. He is currently taking simvastatin (Zocor), aspirin, lisinopril (Prinivil, Zestril), and hydrochlorothiazide. His last LDL-cholesterol level was 70 mg/dL and his blood pressure is 130/80 mm Hg.

Which one of the following additions to his current regimen would be most appropriate at this time?

A) Amlodipine (Norvasc) B) Diltiazem (Cardizem) C) Verapamil (Calan, Verelan) D) Metoprolol (Lopressor, Toprol-XL) E) No changes

19 68. You see a 55-year-old female for the first time. She has a 2-year history of chronic daily cough; thick, malodorous sputum; and occasional hemoptysis. She has been treated with antibiotics for recurrent respiratory infections, but is frustrated with her continued symptoms. She has never smoked. Her FEV1/FVC ratio is 60% and CT shows bronchial wall thickening and luminal dilation.

The most likely diagnosis is

A) emphysema B) bronchiectasis C) chronic bronchitis D) bronchiolitis E) asthma

69. Which one of the following is a classic finding in multiple myeloma?

A) Hypokalemia B) Bone pain C) D) Hepatic failure E) Insomnia

70. Which one of the following is the recommended duration of thromboprophylaxis following total hip arthroplasty, starting from the day of surgery and including outpatient prophylaxis?

A) 7 days B) 14 days C) 35 days D) 60 days E) 90 days

71. A 52-year-old male with hypertension complains of increased dyspnea for the past 6 months. He reports that he has increased fatigue and dyspnea with normal activities. There is no cough or chest pain. He has a 30-pack-year history of smoking.

On examination his blood pressure is 130/85 mm Hg, pulse rate 90 beats/min, respiratory rate 18/min, and O2 saturation 95% on room air. Heart sounds are normal with no murmurs. Auscultation of the lungs reveals bilateral rhonchi.

In addition to ordering a chest radiograph, which one of the following should be performed next in the evaluation of this patient’s dyspnea?

A) A B-type natriuretic peptide (BNP) level B) A D-dimer level C) Arterial blood gas measurement D) Spirometry E) High-resolution CT of the chest

20 72. You respond to a code blue in the obstetrics department. The patient is a 19-year-old primigravida at 35 weeks gestation, hospitalized with severe preeclampsia. A nurse anesthetist has placed an oral airway and is administering 100% oxygen to the apneic patient. She reports no difficulty ventilating the patient with a bag and valve, and no gagging with oral airway insertion. The patient’s blood pressure is 100/60 mm Hg and her pulse rate is 70 beats/min and regular. Her pupils are equal and sluggishly reactive, and she is flaccid and areflexic. The patient had been treated with a magnesium sulfate infusion and a recent bolus of labetalol.

Which one of the following medications should you administer initially?

A) Calcium gluconate B) Fosphenytoin C) Labetalol D) Lorazepam (Ativan) E) Dopamine

73. A 21-year-old male presents with a complaint of headaches for the past 6 months. He has severe, sharp, right-sided periorbital pain 3–4 days each week. When these headaches occur his right eye gets watery, his right nostril feels clogged, and his forehead feels sweaty. When he gets the headaches he takes four 200-mg ibuprofen tablets and goes into a dark, quiet room. The headaches usually resolve in about 90 minutes. Currently he is feeling well and his examination is completely normal.

What type of headache does he most likely have?

A) Medication overuse headache B) Migraine C) Paroxysmal hemicrania D) Temporal arteritis E) Cluster headache

21 74. An 82-year-old white male has a cardiopulmonary arrest while mowing his lawn and his heart rhythm is restored after 8 minutes of CPR by a neighbor. He is now your patient in the coronary care unit. He is on a ventilator and has severe hypoxic encephalopathy. Echocardiography shows an ejection fraction of 12% as a result of the massive anterior myocardial infarction he sustained. Your neurology consultant confirms that the patient will never again be able to meaningfully communicate, and will be ventilator-dependent.

Prior to this, the patient had been living independently and had no health problems. He has no living relatives, and his attorney confirms that he has no written advance directives. The neighbor, who is a close friend, tells you that on several occasions recently he and the patient had discussed such a scenario, and that the patient had said that if he had little chance of a meaningful recovery he would not want to remain on life support.

In consultation with the hospital ethics committee, which one of the following would be most appropriate in this case?

A) Transfer care of the patient to another physician B) Ask a court to appoint a guardian to make medical decisions C) Withdraw life support D) Defer the decision regarding life support to the hospital attorney E) Ask the patient’s attorney to decide whether to terminate life support

75. According to national and international guidelines, which one of the following is the next step for adults with asthma who require therapy with inhaled $-agonists more than three times a week?

A) Inhaled glucocorticoids B) Inhaled salmeterol (Serevent) C) Sustained-release oral $-agonists D) Sustained-release oral theophylline

76. A 30-year-old male presents to your office with a 3-week history of nausea, weight loss, diarrhea, and hematochezia. He states that he has had similar episodes twice in the past and was treated at the local urgent care clinic for infectious diarrhea, with resolution of his symptoms. Your initial laboratory workup is negative for enteric pathogens and you refer the patient for colonoscopy and esophagogastroduodenoscopy with small bowel follow-through. The patient is found to have multiple noncontiguous transmural ulcerations throughout both the small and large intestines.

Which one of the following initial management strategies is most likely to induce remission in this patient?

A) Laparotomy with colectomy B) Metronidazole (Flagyl) C) Prednisone D) Infliximab (Remicade)

22 77. A 7-year-old male presents with a 3-day history of sore throat, hoarseness, fever to 100°F (38°C), and cough. Your examination reveals injection of his tonsils, no exudates, shotty lymphadenopathy, and normal breath sounds.

Which one of the following would be most appropriate?

A) Symptomatic treatment only B) Empiric treatment for streptococcal pharyngitis C) A rapid antigen test for streptococcal pharyngitis D) A throat culture for streptococcal pharyngitis E) An office test for mononucleosis

78. Which one of the following is more common in non-Hispanic whites than in the Hispanic population?

A) Obesity B) Osteoporosis C) Diabetes mellitus D) Hypertension E) Neurocysticercosis

79. A 60-year-old male smoker has lung cancer, and a life expectancy of 4–6 months. His wife is concerned about his state of mind and requests medication for him. His cancer-related pain is generally controlled.

When evaluating the patient, which one of the following features would be more characteristic of depression as opposed to a grief reaction?

A) Insomnia B) Loss of interest or pleasure in all activities C) Feelings of guilt D) Thoughts of wanting to die E) Psychomotor agitation

23 80. A 70-year-old male with long-standing poorly controlled hypertension presents with a 6-month history of fatigue and dyspnea on exertion. He lives in a rural area and continues to be active outdoors but has less stamina. He denies edema, orthopnea, chest pain, and palpitations. His only medication is hydrochlorothiazide.

On examination the patient’s pulse rate is 80 beats/min, his blood pressure is 160/90 mm Hg, and his O2 saturation is 97% on room air. You hear a grade 2/6 midsystolic ejection murmur, which is loudest at the right upper sternal border but is also heard at the lower left sternal border. An S4 is heard at the apex, with the point of maximal intensity at the anterior axillary line of the 5th intercostal space. The S2 is not split but is a single sound. There is no S3. His lungs are clear and there is no peripheral edema.

Which one of the following conditions is the most likely cause of the auscultatory findings?

A) Aortic regurgitation B) Aortic stenosis C) Mitral regurgitation D) Mitral stenosis E) Mitral valve prolapse

81. A 63-year-old male with a history of alcoholism and compensated hepatic cirrhosis asks if there are pain medications he can use to treat his chronic low back pain and knee and hand osteoarthritis. He also has occasional headaches. He has not used alcohol for several years.

Which one of the following medications is CONTRAINDICATED in this patient?

A) Acetaminophen B) Gabapentin (Neurontin) C) Naproxen D) Pregabalin (Lyrica) E) Tramadol (Ultram)

82. A 64-year-old male comes to your office for evaluation of a persistent rash affecting his groin. It is itchy but not painful and does not affect his daily activities. He has tried over-the-counter antifungal creams without relief. On examination you find well-demarcated, dark red patches in the inguinal region bilaterally. When examined with a Wood’s light the area fluoresces coral-red.

The most effective treatment for this condition is topical

A) ketoconazole (Nizoral) B) erythromycin C) hydrocortisone D) mupirocin (Bactroban) E) terbinafine (Lamisil)

24 83. A 58-year-old healthy white female sees you for a routine visit. She is monogamous with her husband, is a nonsmoker, has two alcoholic drinks a week, and has mild GERD. Her BMI is normal. She takes an over-the-counter H2-blocker and a multivitamin with calcium. She had a normal mammogram 1 month ago and a negative colonoscopy at age 53. She has never had a DXA scan or screening for ovarian cancer. Her family history is noncontributory.

According to the U.S. Preventive Services Task Force, you should recommend

A) HIV screening B) CA-125 testing for ovarian cancer screening C) DXA for osteoporosis screening D) colonoscopy for colorectal cancer screening

84. A 27-year-old male complains of difficulty sleeping, forgetfulness, numbness, and feeling detached from life. These symptoms started 2 weeks ago after he was in a motor vehicle accident in which his younger brother was killed. He admits to troubling flashbacks to the accident and would like help with these problems.

Which one of the following is likely to be most effective for this patient?

A) Cognitive-behavioral therapy B) Critical incident stress debriefing C) Supportive counseling D) Fluoxetine (Prozac) E) Propranolol

85. A 24-year-old gravida 2 para 1 at 10 weeks gestation presents with fever, myalgias, headache, and malaise. There have been multiple cases of influenza in the community and her influenza swab is positive.

Which one of the following is recommended by the Centers for Disease Control and Prevention in this situation?

A) Rimantadine (Flumadine) B) Oseltamivir (Tamiflu) C) Acyclovir (Zovirax) D) Supportive therapy only

25 86. A 54-year-old male presents with progressively worsening pain just below his right knee. He describes the pain as deep and aching, and says it is always present throughout the day, even while he is at rest, and worsens at night. Weight bearing intensifies the pain, as does heat. The patient does not recall any injury or other reason for the leg to hurt. He has not had any fever. His family history is positive for osteoarthritis in both parents when they were older, and an uncle has had a knee replacement.

A physical examination is negative except for some varus deformity of the right lower extremity just below the knee. There is no redness. Radiographs demonstrate mild to moderate bowing of the proximal tibia. His alkaline phosphatase level is elevated but his (-glutamyl transaminase level is normal. The remainder of a comprehensive metabolic panel is also normal. A CBC is normal, including the WBC count and differential.

You should suspect which one of the following conditions?

A) Osteoarthritis B) Osteoporosis C) Osteomyelitis D) Paget’s disease of bone E) Seronegative spondyloarthritis

87. A 62-year-old male with a 20-year history of diabetes mellitus presents with bilateral calf and buttock pain that occurs after he walks 2 blocks. The symptoms are relieved with rest. On examination his pedal pulses are not palpable and his ankle-brachial index is 1.45.

Which one of the following would be most appropriate?

A) Reassuring the patient that his ankle-brachial index is normal B) MRI of the lumbar spine C) A repeat evaluation in 6 months if the symptoms persist D) MR or CT angiography of the lower extremities

88. An 85-year-old navy veteran presents to your office with a complaint of cough and dyspnea with exertion. He spent his entire career in ship maintenance and repair, and retired from the navy at the age of 45. His chest radiograph shows pleural plaques. He has a 20-pack-year smoking history, but quit at the age of 39.

You suspect his problem is due to occupational exposure to which one of the following?

A) Asbestos B) Beryllium C) Iron oxide D) Silica E) Uranium

26 89. A 15-year-old male is seen in the office for ankle pain. While playing basketball he jumped and landed on the lateral edge of his foot. He had immediate pain and did not continue playing, but was able to walk after the injury. On examination his right ankle has tenderness, swelling, and bruising over the anterior talofibular and calcaneofibular ligaments. There is no bony tenderness.

Which one of the following would be most appropriate at this point?

A) Taping the ankle for future sports participation B) An elastic compression wrap C) A lace-up ankle support D) A radiograph of the ankle E) A below the knee cast

90. Which one of the following is best for preventing acute mountain sickness?

A) Acetazolamide (Diamox Sequels) started the day before arriving at altitude B) Prednisone started the day before arriving at altitude C) Moderate alcohol consumption on the first day at altitude D) Ascending quickly, then resting to acclimatize before beginning planned activities

91. A 70-year-old alcoholic male is recovering from a myocardial infarction. On the fourth hospital day he complains of a sudden onset of excruciating abdominal pain that is not significantly reduced by large doses of morphine. He becomes nauseated, begins to vomit, and has diarrhea.

The patient appears agitated and confused, and his heart rate increases. He also becomes hypotensive. Physical examination of his abdomen reveals minimal tenderness, decreased bowel sounds, and a moderately enlarged liver.

Laboratory Findings

WBCs...... 17,600/mm3 with a left shift (N 4300–10,800) BUN...... 40 mg/dL (N 8–25) Creatinine...... 1.0 mg/dL (N 0.6–1.5) Serum lipase...... 150 U/L (N 0–160) Arterial pH...... 7.14 (N 7.35–7.45)

The most likely diagnosis is

A) alcohol withdrawal syndrome B) pulmonary embolus C) pancreatitis D) acute mesenteric artery embolism E) perforated gastric ulcer

27 92. Which one of the following cardiovascular changes is a recognized age-related effect?

A) Decreased maximal heart rate with exercise B) Decreased myocardial collagen C) Decreased myocardial mass D) Increased left ventricular compliance E) Increased heart rate at rest

93. A 55-year-old male has New York Heart Association class III chronic systolic heart failure due to hypertensive cardiomyopathy. Which one of the following is CONTRAINDICATED in this patient?

A) Carvedilol (Coreg) B) Digoxin C) Ramipril (Altace) D) Spironolactone (Aldactone) E) Verapamil (Calan)

94. A 13-year-old female is brought to your office for evaluation of school difficulties and depressed mood. Her mother and older sister have both been diagnosed with depression. After a thorough history and physical examination, you diagnose major depressive disorder. You arrange for the patient to receive cognitive-behavioral therapy, but after 6 weeks her condition is only minimally improved.

Which one of the following medications would be appropriate to add to this patient’s treatment plan at this point?

A) Fluoxetine (Prozac) B) Imipramine (Tofranil) C) Lithium D) Venlafaxine

95. Which one of the following medications used in the treatment of osteoporosis can also be used to treat the pain associated with acute and chronic vertebral compression fractures?

A) Calcitonin-salmon (Miacalcin) B) Raloxifene (Evista) C) Risedronate (Actonel) D) Teriparatide (Forteo) E) Zoledronic acid (Reclast)

28 96. A morbidly obese 68-year-old male complains of breast enlargement. He has not noticed any pain or discomfort from this problem. His past medical history is negative except for type 2 diabetes mellitus and hypertension. His medications include metformin (Glucophage), 1000 mg twice daily; lisinopril (Prinivil, Zestril), 20 mg daily; and aspirin, 81 mg daily. His family history is negative for breast cancer. A physical examination is negative except for a BMI of 45 kg/m2 and symmetric bilateral adipose tissue in the breast region on inspection and palpation. There is no glandular tissue on careful palpation of the area beneath the areolae and nipples. No nodules or axillary nodes are detected. There is no nipple retraction or discharge, and no skin changes.

Which one of the following is the most likely cause of this problem?

A) Fat necrosis B) Gynecomastia C) Pseudogynecomastia D) Breast cancer E)

97. Which one of the following is most likely to cause hypoglycemia in elderly patients?

A) Metformin (Glucophage) B) Pioglitazone (Actos) C) Glipizide (Glucotrol) D) Sitagliptin (Januvia) E) Glyburide (DiaBeta)

98. A 42-year-old female is troubled by her lack of interest in sex. She is generally healthy, takes no medications, and has regular menstrual periods. She is content with the emotional intimacy of her marriage and has had satisfying sexual interactions in the past. She does not have any religious or cultural barriers regarding her sexuality, and asks for ideas on how to improve her situation.

Which one of the following has consistent evidence of benefit in cases such as this?

A) Cognitive-behavioral therapy B) Viewing pornography C) Oral estrogen D) Oral sildenafil (Viagra) E) Topical testosterone

29 99. A 25-year-old white male truck driver presents with a 1-day history of throbbing rectal pain. Your examination shows a large thrombosed external hemorrhoid.

Which one of the following is the preferred initial treatment for this patient?

A) Infrared coagulation B) Rubber band ligation of the hemorrhoid C) Elliptical excision of the thrombosed hemorrhoid D) Stool softeners and a topical analgesic/hydrocortisone cream

100. A 20-year-old male presents with complaints of abdominal pain and diarrhea. He says he often has abdominal cramping that is relieved with defecation. The pain is accompanied by frequent loose, mucous stools, and his symptoms tend to get worse with stress. He says he has tried antidiarrheal medications and antispasmodics, but did not get satisfactory results.

Your evaluation leads to a diagnosis of diarrhea-predominant irritable bowel syndrome. Which one of the following would be the most appropriate treatment?

A) Fiber supplements B) Neomycin C) Citalopram (Celexa) D) Alosetron (Lotronex) E) Lubiprostone (Amitiza)

101. A 25-year-old male has developed a painless ulcer on the glans of his penis. After an appropriate examination and testing you diagnose primary syphilis and treat him with 2.4 million units of benzathine penicillin intramuscularly in a single dose. Eight hours later, while you are working the evening clinic, he returns because he has a fever of 100.6°F and a bad headache, which he rarely gets. He says he “aches all over.”

Which one of the following would be most appropriate at this time?

A) Three blood cultures from different sites at 30-minute intervals B) CT of the head C) A lumbar puncture D) Doxycycline, 100 mg orally twice a day for 14 days E) Reassurance and antipyretics

102. According to the American Diabetes Association, screening for diabetes mellitus in the asymptomatic patient with no risk factors should begin at which age?

A) 25 years B) 30 years C) 35 years D) 40 years E) 45 years

30 103. A 44-year-old female with localized breast cancer is receiving counseling about adjuvant long-term therapy. Which one of the following is more likely to occur with an aromatase inhibitor such as letrozole (Femara) than with tamoxifen (Soltamox)?

A) Endometrial cancer B) Venous thromboembolism C) Inflammatory arthritis D) Myalgias

104. A 78-year-old asymptomatic male is found to have a platelet count of 90,000/mm3 (N 150,000–300,000) and a slightly decreased WBC count. Which one of the following would be most consistent with a diagnosis of myelodysplastic syndrome?

A) A normal RBC count and indices B) Normocytic anemia C) Microcytic anemia D) Macrocytic anemia E) Polycythemia

105. A 12-year-old female with asthma sees you for a follow-up visit. The girl’s mother states that she is currently coughing several days per week and uses her albuterol (Proventil, Ventolin) inhaler 3–4 times weekly. She has awakened with a cough during the night 3 times in the last month. The patient thinks her asthma only mildly affects her day-to-day activity. In-office spirometry reveals that her FEV1 is 83% of predicted, with a normal FEV1/FVC ratio.

Which one of the following asthma classifications best fits this patient’s presentation?

A) Intermittent B) Mild persistent C) Moderate persistent D) Severe persistent E) Status asthmaticus

106. Which class of medication is first-line therapy for uncomplicated depression during pregnancy?

A) Monoamine oxidase inhibitors (MAOIs) B) SSRIs C) SNRIs D) Stimulants E) Tricyclic antidepressants

31 107. In 2009, the Centers for Medicare & Medicaid Services introduced the concept of Accountable Care Organizations primarily to accomplish which one of the following objectives?

A) Prevent rates of reimbursement from growing faster than GDP B) Increase reimbursement to medically disadvantaged regions of the United States C) Require uninsured individuals to purchase health insurance at fair-market prices D) Deliver more efficient, high-quality services by encouraging cooperation between health care providers E) Reduce the power of integrated health systems to behave as monopolies

108. A 57-year-old male comes to the emergency department after several episodes of vomiting preceded by moderately severe epigastric pain. He says the vomitus looked like coffee grounds. He tells you he has had “heartburn” in the past that was sometimes severe, and occasionally associated with vomiting, but these episodes were almost always relieved by oral antacids. This problem was exacerbated recently after he began taking ketorolac for moderate arthritic pain in his knees and hands. His past medical history and a review of systems reveal no major comorbid disorders.

The patient’s blood pressure is 125/82 mm Hg and his heart rate is 95 beats/min with no signs of shock. His hemoglobin level is 9.5 g/dL (N 13.0–18.0). He is admitted to the hospital and placed on a proton pump inhibitor (PPI) infusion. Upper gastrointestinal endoscopy performed within 3 hours of admission shows no blood in the upper gastrointestinal tract, but reveals a Mallory-Weiss tear and a stomach ulcer containing a dark spot in an otherwise clear base.

Management at this time should include which one of the following?

A) Transfusion with whole blood B) Repeat endoscopy within 24 hours C) Arteriography D) Continued in-hospital observation for at least 72 hours E) Discharge from the hospital on oral PPI therapy

109. A 22-year-old male who is training for a marathon presents with a 2-week history of ankle pain. There is no history of trauma to the ankle. Your examination reveals tenderness over the distal tibia and you suspect a stress fracture.

The initial imaging study of choice for this patient’s ankle is

A) a plain radiograph B) ultrasonography C) CT D) MRI E) a radionuclide bone scan

32 110. A 12-month-old male is brought to your office for a routine checkup and immunizations. He has not received medical care since his 4-month well child visit and has had no immunizations since that time.

Which one of the following vaccines is NOT indicated for this patient?

A) Varicella vaccine B) Rotavirus vaccine C) Hepatitis B vaccine D) MMR vaccine

111. A 69-year-old male with type 2 diabetes mellitus comes to your office for a routine follow-up visit. He takes insulin glargine (Lantus) as a basal insulin, with meal-time boluses of insulin lispro (Humalog). He reports repeated episodes of hypoglycemia with blood glucose levels in the 40–50 mg/dL range. He treats them appropriately by consuming about 15 g of carbohydrates. He has a history of severe episodes of hypoglycemia requiring emergency services. In addition to insulin, his current medications include simvastatin (Zocor), lisinopril (Prinivil, Zestril), and aspirin. He has both diabetic autonomic neuropathy and retinopathy.

Which one of the following factors in this patient’s case is the most significant predictor of severe hypoglycemia?

A) His age B) His sex C) His medications D) His previous episodes of severe hypoglycemia E) His diabetic autonomic neuropathy

112. When compared with an occiput anterior fetal position, a persistent occiput posterior fetal position is less likely to result in

A) spontaneous vaginal delivery B) assisted vaginal delivery C) cesarean delivery D) a third or fourth degree perineal laceration E) excessive maternal blood loss

33 113. One year after being diagnosed with early Alzheimer’s disease, one of your long-time patients develops symptomatic carotid stenosis. A vascular surgeon has recommended surgical treatment, but the patient’s family is uncertain whether he should have the surgery and if he is capable of making the decision. The children are evenly split in their opinion regarding the surgery, and they ask for your input.

Which one of the following is true regarding this situation?

A) The patient is incapable of making this decision because of his dementia B) The Mini-Mental State Examination score determines competence C) The patient should be evaluated by a psychiatrist D) A judicial determination of competence should be obtained E) The patient’s decision-making capacity can be adequately assessed by clinical evaluation

114. A 70-year-old white female asks you to evaluate her right shoulder because of pain and limited range of motion. Further history reveals that 2 months ago she slipped in her kitchen and caught onto the refrigerator door handle to avoid falling to the floor. On examination she has pain and weakness at 45° of abduction and weakness on external rotation.

She should be treated for

A) bicipital tendinitis B) disruption of the glenoid fossa C) rotator cuff tear D) acromioclavicular separation E) incomplete fracture of the humeral head

115. Which one of the following vesiculobullous diseases is associated with gluten sensitivity?

A) Bullous pemphigoid B) Herpes gestationis C) Erythema multiforme D) Porphyria cutanea tarda E) Dermatitis herpetiformis

116. A 32-year-old male comes to your office because of wrist pain following a fall on the ice 10 days ago. Examination of the wrist shows no deformity or swelling, but extension is decreased and he has diffuse tenderness over the dorsum of the wrist, particularly just distal and dorsal to the radial styloid. A radiograph is shown below.

Which one of the following does the radiograph show?

A) A hamate fracture B) A scaphoid fracture C) A lunate dislocation D) A scapholunate dislocation

34 117. An 18-year-old basketball player comes to your office for evaluation of finger pain. During a basketball game yesterday he was hit on the tip of his right second digit and now has finger pain and difficulty moving his finger. On examination he has bruising and tenderness over the distal interphalangeal (DIP) joint. His DIP joint is in the flexed position and he is unable to extend the joint. A radiograph shows a fracture at the dorsal surface of the proximal distal phalanx involving 10% of the joint space.

What is the most appropriate management of this injury?

A) Taping the finger to the adjacent finger B) Splinting in full extension C) Splinting in 45° of flexion D) Urgent surgical management E) Intermittent splinting for comfort

118. A 31-year-old gravida 2 para 2 sees you for a routine annual visit. Her Papanicolaou (Pap) test is normal and high-risk HPV testing is negative. She has never had an abnormal Pap test.

According to the guidelines of the U.S. Preventive Services Task Force and the American College of Obstetricians and Gynecologists, this patient’s cervical cytology and HPV testing should be repeated in

A) 1 year B) 2 years C) 3 years D) 4 years E) 5 years

119. A 59-year-old male college professor presents with a 2-month history of right medial knee pain. There is no history of injury or overuse. He has no other specific joint pain except for occasional myalgias and arthralgias of his legs and arms. On most days he has morning stiffness lasting 15–20 minutes after getting out of bed. A review of systems is otherwise negative.

On examination the right knee has full range of motion. There is tenderness at the medial joint line, but no clicking or ligamentous instability. There is crepitus with movement in both knees.

Which one of the following diagnostic tests would be most appropriate at this time?

A) Serologic testing B) Synovial fluid analysis C) Plain radiography D) MRI without contrast E) MRI with contrast

35 120. An 87-year-old female is brought to the emergency department after losing consciousness at the dinner table. Her history indicates recent unintentional weight loss. Further evaluation ultimately reveals a large mass at the head of the pancreas and extensive metastasis to numerous organs, including the brain. Her life expectancy is estimated to be 2–3 weeks. The patient chooses to receive hospice care but becomes very depressed.

Which one of the following would be best for improving her depression?

A) Electroconvulsive therapy B) Methylphenidate (Ritalin) C) Mirtazapine (Remeron) D) Fluoxetine (Prozac) E) Nortriptyline (Pamelor)

121. A 40-year-old male presents with a sudden onset of unilateral peripheral facial nerve weakness 1 hour ago. Which one of the following is most likely to shorten his symptoms?

A) Corticosteroid therapy B) Antiviral therapy C) Thrombolytic therapy D) Hyperbaric oxygen therapy E) Facial nerve decompression

122. You see a 16-year-old white female for a preparticipation evaluation for volleyball. She is 183 cm (72 in) tall, and her arm span is greater than her height. She wears contacts for myopia.

Which one of the following should be performed at this time?

A) An EKG B) Echocardiography C) A stress test D) A chest radiograph E) Coronary MR angiography

123. An 80-year-old male nonsmoker with Parkinson’s disease is treated for community-acquired pneumonia with azithromycin (Zithromax), 500 mg/day for 10 days. On follow-up the patient feels better but still has a productive cough. A repeat chest radiograph reveals a single thin-walled cavity lesion in the left lower lobe.

It would be most appropriate to replace this patient’s azithromycin with

A) doxycycline B) clindamycin (Cleocin) C) metronidazole (Flagyl) D) trimethoprim/sulfamethoxazole (Bactrim, Septra)

36 124. The diagnosis of delirium is based on

A) the history and physical findings B) complete metabolic panel results C) toxicology screening results D) EEG findings E) MRI of the brain

125. A 55-year-old female receives a gynecologic and breast examination from a nurse practitioner, who also orders a routine mammogram. Who is legally responsible for ensuring that the patient is notified of the results of the mammogram?

A) The nurse practitioner B) The supervising physician C) The facility performing the mammogram D) The patient

126. Which one of the following is most suggestive of plantar fasciitis?

A) Heel pain at rest B) Lateral heel tenderness with palpation C) A heel spur on radiographs D) Prompt relief with NSAIDs E) Heel pain that is worse with the first steps in the morning

127. A 72-year-old male is brought to your office by a friend because of increasing confusion, irritability, and difficulty walking. This began shortly after the patient’s car broke down in a rural area and he had to walk a mile to get to a phone and call the friend. The temperature outdoors has been near 100°F.

On examination the patient has a rectal temperature of 39.5°C (103.1°F), a pulse rate of 110 beats/min, and a blood pressure of 100/60 mm Hg. His shirt is still damp with sweat.

Which one of this patient’s findings indicates that he has heatstroke rather than heat exhaustion?

A) Confusion B) Sweating C) His temperature D) His heart rate E) His blood pressure

37 128. A 42-year-old male sees you for help to quit smoking. His sister had excellent results with bupropion (Zyban) and he asks if he could try using it.

When you review his medical history, which one of the following would be a contraindication to bupropion?

A) Diabetes mellitus B) Gout C) Hypertension D) Hyperthyroidism E) A seizure disorder

129. A 20-year-old female distance runner presents with a 1-month history of left knee pain. The pain is worse with her first few steps in the morning and when going down stairs. Examination of the knee reveals no deformity, effusion, or ligamentous laxity. The knee joint and surrounding tissues are not tender to palpation, with the exception of an area 2 cm proximal to the left lateral joint line.

What is the most likely cause of this patient’s pain?

A) Osteoarthritis of the knee joint B) Pes anserine bursitis C) Iliotibial band syndrome D) Chronic lateral meniscal tear E) Osgood-Schlatter disease

130. A 70-year-old retired minister presents with fatigue. Two weeks ago he returned from a mission trip to Mexico and had a 4-day episode of watery diarrhea, which resolved with bismuth subsalicylate (Pepto-Bismol) treatment. He has a 5-year history of diabetes mellitus and hypertension. Six months ago a myocardial infarction was treated successfully with a drug-eluting stent. His blood pressure is 138/80 mm Hg, pulse rate 86 beats/min, respiratory rate 18/min, and temperature 36.9°C (98.4°F). The remainder of the physical examination is unremarkable.

Laboratory Findings

Hemoglobin...... 14.2 g/dL (N 13.0–18.0) WBCs...... 8000/mm3 (N 4300–10,800) BUN...... 20 mg/dL (N 8–25) Creatinine...... 1.9 mg/dL (N 0.6–1.5) Potassium...... 3.9 mEq/L (N 3.4–4.8) Glucose...... 140 mg/dL AST (SGOT)...... 30 U/L (N 10–55) Creatine kinase...... 40 U/L (N 60–400)

38 Which one of this patient’s medications should be stopped?

A) Carvedilol (Coreg) B) Clopidogrel (Plavix) C) Metformin (Glucophage) D) Simvastatin (Zocor) E) Sitagliptin (Januvia)

131. An 82-year-old female with terminal breast cancer has been admitted to hospice care. She is having severe pain that you will manage with opioids.

Which one of the following would be appropriate to recommend for preventing constipation?

A) Fiber supplements B) Docusate (Colace) C) Metoclopramide (Reglan) D) Polyethylene glycol (MiraLax) E) No preventive measures, and treatment only if constipation develops

132. A 19-year-old female presents with a 2-week history of sore throat and mild adenopathy. She denies fever, congestion, and cough. Her review of systems is negative. Rapid tests for streptococcal infection and mononucleosis are negative. A throat swab shows Neisseria gonorrhoeae.

Which one of the following is recommended for this patient?

A) A pelvic examination B) A cervical culture or urine nucleic acid amplification testing C) Cotreatment for Chlamydia infection D) A test of cure in 1 month

39 133. An elderly alcoholic male is brought to the hospital by his grandson, who found him in poor condition. The grandson reports that his family has not seen the patient in months.

The patient says he has no health complaints, but he is obviously malnourished, dirty and unkempt, and mildly intoxicated. You admit the patient to the hospital and begin providing supportive care, including intravenous fluids with dextrose, a regular diet, and physical therapy evaluation. On the evening of the second day he becomes weak and more confused. His blood pressure is 88/56 mm Hg, and he has a seizure. Your evaluation includes the following laboratory findings:

Glucose...... 60 mg/dL (N 70–110) BUN...... 9 mg/dL (N 6–20) Creatinine...... 2.6 mg/dL (N 0.8–1.3) Creatine kinase...... 480 U/L (N 38–174) Troponin I...... <0.1 ng/mL (N<0.6) Albumin...... 2.7 g/dL (N 3.4–4.8) Calcium...... 8.2 mg/dL (N 8.6–10.0) Phosphate...... 0.7 mg/dL (N 2.7–4.5) ALT (SGPT)...... 68 U/L (N 10–40) AST (SGOT)...... 88 U/L (N 10–30)

This episode is most likely related to abnormal levels of which one of the following?

A) Glucose B) Creatinine C) Creatine kinase D) Calcium E) Phosphate

134. The mother of a 6-month-old male tells you that he sometimes wheezes while feeding, and this is occasionally associated with a cough. Changing his position does not help.

Which one of the following is the most likely diagnosis?

A) Tracheoesophageal fistula B) Laryngeal cleft C) Gastroesophageal reflux disease D) Foreign body aspiration E) Tracheomalacia

135. A 42-year-old female has a 3-day history of an intensely pruritic rash on her arm, shown below. Which one of the following is most likely to have caused these skin lesions?

A) Balsam of Peru B) Bedbugs C) Neomycin D) Nickel E) Poison ivy

40 136. Which one of the following activities is most likely to be impaired in early dementia?

A) Dressing B) Eating C) Toileting D) Grooming E) Cooking

137. A 3-year-old female is brought to your office for evaluation of mild intoeing. The child’s patellae face forward, and her feet point slightly inward.

Which one of the following would be most appropriate?

A) Reassurance and continued observation B) Foot stretching exercises C) Orthotics D) Night splints E) Surgery

138. Residents of long-term care facilities who have advance directives are more likely to

A) be placed on a ventilator B) undergo feeding tube placement C) enroll in hospice care D) be admitted to a hospital E) die in a hospital

139. A 70-year-old retired farmer presents with an angulated right knee and a painful hip. He asks you about the possibility of having knee replacement surgery, although he is not eager to do so.

You would advise him that the major indication for knee replacement is

A) severe joint pain B) marked joint space narrowing on radiologic studies C) destruction and loss of motion of the contralateral joint D) an acutely infected joint

140. The audiogram shown below depicts which type of bilateral hearing loss?

A) Conductive hearing loss due to cerumen impaction B) Conductive hearing loss due to perforated tympanic membranes C) Noise-induced sensorineural hearing loss D) Sensorineural hearing loss due to Meniere’s disease E) Sensorineural hearing loss due to a vestibular schwannoma

41 141. A 15-year-old male has a 1-week history of a nonproductive cough, a low-grade fever, a sore throat, and hoarseness. His respiratory rate is 22/min but unlabored, his temperature is 38.1°C (100.6°F), and his O2 saturation is 94% on room air. A chest radiograph reveals bilateral interstitial infiltrates.

Which one of the following treatments would be most appropriate for this patient?

A) Ceftriaxone (Rocephin) B) Amoxicillin C) Cefdinir D) Linezolid (Zyvox) E) Azithromycin (Zithromax)

142. Which one of the following is associated with a herald patch?

A) Pityriasis alba B) Pityriasis lichenoides C) Pityriasis rosea D) Pityriasis rubra pilaris E) Pityriasis (tinea) versicolor

143. A patient who has terminal metastatic lung cancer with bony metastases is being cared for at home and using hospice services. The hospice nurse calls you during the night because the family had called her to come to the house. When she arrived she found the patient acutely agitated, confused, and disoriented, and he does not recognize his family members. The patient is trying to hit his caretakers, who are distressed by the situation.

In addition to checking for underlying causes of these acute symptoms, which one of the following is most appropriate for managing this problem?

A) Amitriptyline B) Haloperidol C) Scopolamine D) Trazodone (Oleptro)

144. A 55-year-old male with a 4-year history of type 2 diabetes mellitus was noted to have microalbuminuria 6 months ago, and returns for a follow-up visit. He has been on an ACE inhibitor and his blood pressure is 140/90 mm Hg.

The addition of which one of the following medications would INCREASE the likelihood that dialysis would become necessary?

A) Hydrochlorothiazide B) Amlodipine (Norvasc) C) Atenolol (Tenormin) D) Clonidine (Catapres) E) Losartan (Cozaar)

42 145. A 30-year-old female presents to your office with a clear nasal discharge, sneezing, nasal congestion, and nasal itching. She notes that these symptoms generally occur in the spring and fall.

The most effective drug for treatment and prevention is

A) cetirizine (Zyrtec) B) cromolyn nasal spray (NasalCrom) C) ipratropium nasal spray (Atrovent) D) montelukast (Singulair) E) fluticasone nasal spray (Flonase)

146. A 45-year-old obtunded male is brought to the emergency department by ambulance. Slow, shallow respirations are noted. His wife tells you that he is being treated by a local pain specialist for chronic back pain stemming from a severe workplace injury 2 years ago. A urine immunoassay drug screen is negative for opioids.

Which one of the following opioid medications would NOT be detected by this drug screen?

A) Codeine B) Fentanyl C) Hydrocodone D) Hydromorphone (Dilaudid) E) Morphine

147. A 48-year-old female presents as a new patient to your office. She has not seen a physician for several years and her medical history is unknown. Her BMI is 24.4 kg/m2 and she is not taking any medication. Her blood pressure is 172/110 mm Hg in the left arm sitting and 176/114 mm Hg in the right arm sitting; her cardiovascular examination is otherwise unremarkable. A baseline metabolic panel reveals a creatinine level of 0.68 mg/dL (N 0.6–1.1) and a potassium level of 3.3 mEq/L (N 3.5–5.5).

If the patient’s hypertension should prove refractory to treatment, which one of the following tests is most likely to reveal the cause of her secondary hypertension?

A) A 24-hour urine catecholamine level B) A plasma aldosterone/renin ratio C) MRA of the renal arteries D) Echocardiography E) A sleep study (polysomnography)

43 148. Which one of the following medications used to treat psychiatric disorders is associated with an increased risk of agranulocytosis?

A) Carbamazepine (Tegretol) B) Lithium C) Aripiprazole (Abilify) D) Olanzapine (Zyprexa) E) Imipramine (Tofranil)

149. Which one of the following is considered first-line therapy for mild to moderate Alzheimer’s disease?

A) Donepezil (Aricept) B) Memantine (Namenda) C) Selegiline (Eldepryl) D) Risperidone (Risperdal) E) Ginkgo biloba

150. A 35-year-old white female complains of severe pain in her right shoulder. She notes that at night the pain intensifies if she rolls onto her right side. She has marked pain with no weakness on abduction of the shoulder; range of motion of the shoulder is normal.

The most likely cause of her pain is

A) bicipital tendinitis B) complete rotator cuff tear C) osteoarthritis of the shoulder D) subacromial bursitis E) frozen shoulder

151. Static stretching before running has been shown to

A) increase strength B) increase endurance C) decrease the frequency of lower limb muscle injury D) reduce delayed-onset muscle soreness E) have no benefit

44 152. A 17-year-old football player presents to your office on a Monday with a right knee injury. He injured the knee in Friday night’s game when an opposing player fell against the knee from the front while the patient had his right foot planted. He was unable to bear weight after the injury, and noted immediate swelling of the knee.

A positive result with which one of the following would indicate an anterior cruciate ligament tear?

A) Ballottement test B) Lachman test C) McMurray test D) Posterior drawer test E) Thessaly test

153. A 14-year-old female sees you for a well child visit. She is healthy and has no complaints or concerns today. A review of her past immunizations shows that she was up to date on all required immunizations at her 8-year-old well child visit. She also received HPV vaccine at age 11 and 12, and quadrivalent meningococcal vaccine (MCV4) and TdaP at age 12.

Which one of the following vaccines should she receive at this visit?

A) Hepatitis C B) HPV C) Inactivated poliovirus D) Measles E) Rubella

154. A 79-year-old male with low libido has been found to have a low testosterone level. Which one of the following studies is important to obtain prior to treatment with testosterone replacement?

A) Hematocrit/hemoglobin B) An FSH level C) Hemoglobin A1c D) A basic metabolic profile

155. You are counseling a 45-year-old obese male regarding weight loss. The patient has elevated triglycerides, low HDL-cholesterol, and stage 1 hypertension. He does not currently take medications and would like to avoid taking medications in the future. The patient has heard good things about low-carbohydrate diets and asks your opinion.

A low-carbohydrate diet in a patient such as this is most likely to result in

A) increased LDL-cholesterol B) increased triglycerides C) increased blood pressure D) development of metabolic syndrome E) better short-term weight loss than with a low-fat diet

45 156. A 24-year-old nulligravida comes to your office for contraception counseling. She has a seizure disorder that is well controlled on carbamazepine (Tegretol). She is a nonsmoker and has no other medical problems or complaints. She is currently in a relationship and does not want to get pregnant in the next several years.

Which one of the following contraceptive options would be the most appropriate?

A) Progestin-only pills B) Combined oral contraceptives C) The etonogestrel/ethinyl estradiol vaginal ring (NuvaRing) D) The norelgestromin/ethinyl estradiol contraceptive patch (Ortho Evra) E) A levonorgestrel intrauterine device (Mirena)

157. A 78-year-old male is on dual antiplatelet therapy (aspirin and clopidogrel) as a result of a stroke 6 months ago. He recently underwent coronary angiography, and his cardiologist has scheduled coronary artery bypass surgery for a week from today.

Which one of the following is recommended with regard to his antiplatelet therapy?

A) Stopping only aspirin 5 days before surgery B) Stopping only clopidogrel 5 days before surgery C) Stopping both aspirin and clopidogrel 5 days before surgery D) Continuing both aspirin and clopidogrel

158. A 72-year-old female presents to the emergency department complaining of dizziness. She also reports palpitations since yesterday, with dyspnea on exertion. She does not have chest pain. On examination her blood pressure is 102/60 mm Hg, pulse rate 140 beats/min, respirations 16/min, and O2 saturation 94% on room air. She has tachycardia with no murmurs and her lungs are clear. The remainder of her examination is normal, including the absence of lower-extremity edema. Her EKG is shown below.

Which one of the following is the best initial step in the management of her tachycardia?

A) Amiodarone (Cordarone) B) Intravenous adenosine (Adenocard) C) Intravenous metoprolol tartrate (Lopressor) D) Immediate synchronized cardioversion E) Unsynchronized cardioversion under conscious sedation

159. Misleadingly low serum sodium can be caused by

A) hyperglycemia B) diuretic use C) heart failure D) renal disease

46 160. The parents of a 5-year-old male ask you about treating him for attention-deficit/hyperactivity disorder (ADHD) because of his hyperactivity at home and preschool. According to the newest guidelines, the most appropriate next step is to

A) prescribe a very low dose of stimulant medication B) explain to the parents that drug therapy for ADHD is not appropriate at this age C) perform a dietary history focusing on the child’s sugar intake D) explore the nature of his hyperactivity and whether there are coexisting behavioral problems

161. A 17-year-old female presents to your office with anterior knee pain. She tells you she recently started a running program. She says the pain is worse running down hills, and is vaguely localized just medial to the patella. Examination of the knee shows no effusion or instability, and there is no joint-line pain or patellar tenderness. McMurray’s maneuver is negative. Plain radiographs of the knee appear normal.

Which one of the following would be most appropriate at this point?

A) MRI of the knee B) Modification of her running program and a quadriceps and hip strengthening program C) Static stretching of the quadriceps and hamstrings prior to running D) A corticosteroid injection in the area of the pes anserine bursa

162. A 50-year-old female reports a 1-month history of pain in her wrists. She does not recall any injury. On examination both wrists are warm but not red, feel boggy on palpation, and lack 30° of both flexion and extension. No other joints are affected. She feels fatigued and unwell, but attributes this to her busy schedule.

Radiographs of the wrists are normal. Laboratory findings are unremarkable except for a mildly elevated erythrocyte sedimentation rate and a negative rheumatoid factor.

Which one of the following is the most likely diagnosis?

A) Rheumatoid arthritis B) Osteoarthritis C) Inapparent injury D) Fibromyalgia E) Lyme disease

47 163. During a well child examination, you notice that a 2-month-old male has a flattened left occiput. His records show that his skull was normally shaped at birth. Further evaluation shows that the left frontal region is more prominent than the right, and the left ear is slightly forward of its expected position. The infant seems comfortable rotating his head to either side while being held in his mother’s arms.

Which one of the following would be appropriate at this time?

A) Recommend that the infant sleep in a prone position, and follow up in 1 month B) Educate the parents about positioning and follow up in 2 months C) Order physical therapy D) Order CT of the head E) Refer for surgical evaluation

164. A 25-year-old white female comes to your office complaining of abdominal pain. She requests that you hospitalize her and do whatever is necessary to get rid of the pain that has been present for a number of years. She has difficulty describing the pain. She is a divorced single parent, and becomes defensive when asked about her former marriage, stating only that her ex-husband is an alcoholic, “just like my father.”

Her previous medical history includes an appendectomy, a cholecystectomy, and a hysterectomy. On physical examination she appears healthy, and a CBC, erythrocyte sedimentation rate, serum amylase level, and comprehensive metabolic panel are all normal.

Management of this patient should include which one of the following?

A) Reassurance that her symptoms are simply psychogenic B) Long-term use of antidepressants C) Scheduling frequent, regular office visits D) Hospitalization, then consultation with a psychiatrist E) Referral to a surgeon for exploratory laparotomy

165. A 25-year-old clinically healthy African-American female is involved in a minor motor vehicle collision. Chest radiographs obtained after the accident reveal bilateral hilar lymphadenopathy. She has no history of environmental exposures and has no symptoms. A physical examination is completely normal. Your initial workup includes a normal comprehensive metabolic panel, CBC, and urinalysis; a negative tuberculin skin test; a normal EKG; and normal pulmonary function tests. A transbronchial lung biopsy specimen reveals a noncaseating epithelioid granuloma.

Which one of the following would be the most appropriate treatment at this time?

A) Long-term high-dose systemic corticosteroids B) Pulsed doses of systemic corticosteroids C) Inhaled corticosteroids D) Oral methotrexate (Trexall) weekly E) Observation only

48 166. A 68-year-old white male with severe COPD has diminished symmetric breath sounds, +1 ankle edema, a regular heart rhythm, a loud pulmonic component of the second heart sound, and a right parasternal heave. Which one of the following interventions is most likely to be therapeutic?

A) "-Blocker therapy B) ACE inhibitor therapy C) Calcium channel blocker therapy D) Digoxin therapy E) Long-term oxygen therapy

167. A 23-year-old female nonsmoker has a history of an unusually high number of sinus infections and episodes of bronchitis. She has not required hospitalization, and the infections have not been due to a fungus or methicillin-resistant Staphylococcus aureus. Her growth as a child was normal, and she has a BMI of 24 kg/m2.

Which one of the following laboratory findings would be most likely?

A) An abnormal nitroblue tetrazolium test B) Severe lymphopenia C) Thrombocytopenia D) Decreased serum levels of IgG, IgM, and IgA

168. Which one of the following effects of vitamin D is supported by the best evidence?

A) It prevents B) It prevents colon cancer C) It prevents dementia D) It reduces falls in community-dwelling older adults

169. The Timed Up and Go test consists of a patient rising from a chair, walking 3 meters (or about 10 feet), turning around, walking back, and sitting back down. The average healthy adult over the age of 60 can perform this in how many seconds?

A) 5 B) 10 C) 20 D) 30 E) 45

170. What is the specific antidote used to treat methanol poisoning?

A) Ethanol B) Haloperidol C) Lorazepam (Ativan) D) Naloxone E) Thiamine

49 171. A 6-month-old male is brought in for a routine checkup. Only one testicle is palpable. The genital examination is otherwise within normal limits.

Which one of the following would be most appropriate at this time?

A) Observation only, until 18 months of age B) Abdominal ultrasonography C) Urologic referral for surgical exploration D) HCG treatment for 3 months

172. A 72-year-old male is admitted to the hospital after a syncopal episode that led to a skull fracture. All of his blood tests are in the normal range. The following morning his sodium level is 132 mEq/L (N 135–145) and further testing confirms that he is suffering from the syndrome of inappropriate secretion of antidiuretic hormone (SIADH). He is placed on a free-water restriction of <1 L/day. Later that evening he complains of a headache and vomits repeatedly. A recheck of his electrolytes shows that his sodium level has dropped to 121 mEq/L.

What would be the most appropriate way to address his hyponatremia at this time?

A) Start oral tolvaptan (Samsca) B) Start oral sodium tablets C) Start an intravenous infusion of hypertonic saline D) Further restrict fluid intake

173. Human parvovirus B19 is associated with which one of the following?

A) Erythema marginatum B) Erythema multiforme C) Erythema toxicum D) Erythema infectiosum E) Erythema chronicum

174. Which one of the following is the basis for the most effective method of natural family planning?

A) Calendar calculation B) Basal body temperature charting C) Cervical mucus monitoring D) Monitoring for urine estrogen metabolites E) Coitus interruptus (withdrawal)

175. A 32-year-old white primigravida has a at 33 weeks gestation. Which one of the following is the most likely cause?

A) Infection B) Placental disease C) A fetal structural disorder D) A hypertensive disorder

50 176. A 55-year-old female has a 2-year history of a slowly progressive bilateral tremor. The tremor interferes with her writing and eating. She has recently noted head bobbing and a change in her voice.

Which one of the following would be an appropriate first-line medication for this problem?

A) Levetiracetam (Keppra) B) Olanzapine (Zyprexa) C) Alprazolam (Xanax) D) Carbidopa/levodopa (Sinemet) E) Propranolol

177. A 70-year-old male without underlying lung disease presents with a 36-hour history of fever, body aches, cough, and dyspnea. He did not receive influenza vaccine this year, and was recently exposed to his grandson who had influenza.

On examination the patient has a temperature of 38.8°C (101.8°F), a blood pressure of 90/50 mm Hg, a heart rate of 110 beats/min, and an O2 saturation of 87% on room air. A nasal swab rapid antigen test is negative, and his WBC count is 15,000/mm3 (N 4300–10,800). A viral culture is sent to the laboratory. A chest radiograph shows a large lobar pneumonia.

You hospitalize the patient and initiate

A) ceftriaxone (Rocephin) and azithromycin (Zithromax) B) levofloxacin (Levaquin) C) oseltamivir (Tamiflu) D) oseltamivir, ceftriaxone, and azithromycin E) oseltamivir, ceftriaxone, azithromycin, and vancomycin (Vancocin)

178. A mother brings her 10-year-old son to your office because he has recently experienced a flare-up of atopic dermatitis, including increased pruritus. Physical findings include increased erythema of the involved skin on the flexural surfaces of his arms and legs, with weeping eruptions located within areas of lichenification.

Which one of the following topical treatments for managing this episode is supported by the best available evidence?

A) Emollients B) Pimecrolimus (Elidel) C) Mupirocin (Bactroban) D) Corticosteroids E) Antihistamines

51 179. A 34-year-old white female presents with complaints of polydipsia, polyuria, and fatigue. Her family history is positive for diabetes mellitus. On examination she has a BMI of 38 kg/m2. Her fasting blood glucose level is 152 mg/dL and her hemoglobin A1c is 8.5%. Her fasting blood glucose level was 89 mg/dL 6 months ago. She says her vision has been somewhat blurry lately, but she has not noticed any tingling in her feet.

Which one of the following initial management options would be most appropriate?

A) Lifestyle modification B) Pioglitazone (Actos) C) Sitagliptin (Januvia) D) Insulin E) Bariatric surgery

180. Which one of the following is a preferred first-line agent for managing hypertension in patients with stable coronary artery disease?

A) A thiazide diuretic B) An angiotensin receptor blocker C) A $-blocker D) A long-acting calcium channel blocker E) A long-acting nitrate

181. A 54-year-old female presents with a complaint of dizziness. Two days ago, while riding in a friend’s car and trying to read a book, she experienced sudden extreme nausea and a “spinning” feeling that lasted for 20 minutes. She also had a headache that mainly felt like a fullness in the area around her left ear. Since then she has had only mild dizziness when she moves her head too quickly. She recalls experiencing these symptoms on two other occasions but cannot remember the circumstances, although she thinks one episode may have been related to having had too much caffeine.

A review of systems is positive for a humming in her ears over the last few years. On examination both ears appear normal. Mild horizontal nystagmus can be seen on movement of the head to the left. Audiograms are normal in the right ear, with a low-frequency hearing loss on the left.

Which one of the following is the most likely diagnosis?

A) Motion sickness B) Meniere’s disease C) Vestibular migraine D) Benign positional vertigo

52 182. A 32-year-old female has a 3-week history of depressed mood. She reports markedly diminished interest or pleasure in most activities, fatigue, a diminished ability to concentrate, and insomnia. She has had recurrent suicidal thoughts, but has no specific plan. Further investigation reveals a past history of several hypomanic episodes lasting 4–5 days, characterized by a persistently elevated, expansive mood. During these episodes she needed little sleep, was talkative, met multiple goals, and had trouble keeping up with the thoughts that were running through her head. She was treated with lithium in her early twenties but she stopped taking it because it stifled her artistic creativity. She currently takes no medication.

Her physical examination is unremarkable. Results from comprehensive laboratory studies, including a urine toxicology screen, are also normal.

Which one of the following is most appropriate for her current depressive symptoms?

A) Aripiprazole (Abilify) B) Venlafaxine C) Divalproex (Depakote) D) Divalproex and bupropion (Wellbutrin) E) Lithium and paroxetine (Paxil)

183. A 65-year-old male with type 2 diabetes mellitus is having increasing symptoms of angina pectoris. His cardiologist has recommended that he undergo heart catheterization and possible intervention if coronary artery disease is found. He comes to your office prior to the procedure and asks for your thoughts regarding treatment options presented by the cardiologist.

In addition to optimal medical treatment, if this patient is found to have multivessel coronary disease at the time of heart catheterization, you would recommend which one of the following?

A) Angioplasty without stenting B) Angioplasty with bare-metal stents C) Angioplasty with drug-eluting stents D) Angioplasty of the most significantly blocked artery, followed by coronary artery bypass graft surgery E) Coronary artery bypass graft surgery

53 184. A 2½-year-old male is brought to the emergency department with the acute onset of diffuse abdominal pain that began approximately 6 hours ago. He has also had 3 episodes of bilious emesis in the last 2 hours. A review of systems is positive for anorexia today but negative for fever, weight loss, diarrhea, and bloody stools.

On examination the patient’s height and weight are in the 50th percentile for age, his blood pressure is normal, his heart rate is 110 beats/min, and his temperature is 36.9°C (98.4°F). Cardiovascular and pulmonary examinations are unremarkable. The abdominal examination is significant for slightly hypoactive bowel sounds and diffuse tenderness to palpation without rebound, guarding, or rigidity. A genitourinary examination is normal.

Which one of the following studies is the most appropriate next step to diagnose the cause of abdominal pain in this patient?

A) Scrotal ultrasonography B) Abdominal ultrasonography C) Abdominal and pelvic CT D) An upper gastrointestinal series

185. Which one of the following juices can greatly increase the blood level of a statin?

A) Apple B) Grapefruit C) Orange D) Pineapple E) Tomato

186. A 53-year-old male with hypertension, hyperlipidemia, and nonalcoholic fatty liver disease began taking atorvastatin (Lipitor) 3 months ago. His LDL-cholesterol level is now at goal, but he has developed an asymptomatic elevation of his hepatic transaminases to twice-normal levels.

Which one of the following is the most appropriate course of action?

A) Continue the atorvastatin at the current dosage B) Reduce the dosage of atorvastatin by half C) Discontinue atorvastatin and switch to another statin D) Discontinue atorvastatin and switch to an antihyperlipidemic agent from a different class E) Order hepatic ultrasonography

187. A 95% confidence interval means

A) at least 95% of patients with a disease have a positive test for that disease B) at least 95% of patients without a disease have a negative test for that disease C) there is a 95% difference in risk between the treatment and control groups D) it is 95% certain that the true value lies within the given range E) at least 95% of the patients need to receive an intervention instead of the alternative in order for one additional patient to benefit

54 188. An 11-year-old male is brought to your clinic for follow-up after a recent well child visit revealed elevated blood pressure. The parents have restricted his intake of sodium and fatty foods during the last several weeks. His blood pressure today is 140/92 mm Hg, which is similar to the reading at his last visit. The parents checked the child’s blood pressure with a home unit several times and found it consistently to be in the 130s systolic and low 80s diastolic. The child had a normal birth history and has no known chronic medical conditions. Both of his parents and his two younger siblings are healthy. He is at the 75th percentile for both height and weight with a BMI in the normal range. He eats a balanced diet and is active.

What should be the next step for this patient?

A) Reassurance that this is likely white-coat hypertension B) A goal weight loss of at least 5 lb C) Evaluation for causes of secondary hypertension D) Hydrochlorothiazide E) Lisinopril (Prinivil, Zestril)

189. A 52-year-old white male presents for a health maintenance visit. The patient has mild osteoarthritis but is otherwise healthy. He lives at home with his wife. He drinks approximately 2 beers a week and does not smoke. He takes a multivitamin, but no other medications.

What is the recommendation for immunizing this patient with pneumococcal polyvalent-23 vaccine (Pneumovax)?

A) One dose now B) One dose after age 65 C) One dose now, and again after age 65 D) One dose now, in 5 years, and again after age 65 E) No vaccination unless he develops an immunocompromising disease

190. Long-term alleviation of carpal tunnel syndrome in patients with persistent symptoms is best accomplished by which one of the following?

A) Splinting B) Physical therapy C) Ibuprofen D) Corticosteroid injection E) Surgery

55 191. A 32-year-old male presents to an urgent care center with a 2-day history of left calf pain and swelling, which started gradually a few hours after he played tennis. He remembers that he “tweaked” his calf on a serve late in the match but was able to continue playing. He has no history of prior medical problems, and no recent surgery or immobilization.

On examination his left calf appears slightly erythematous and swollen from the mid-calf to the ankle, with 1+ pitting over the lower leg. There is no venous distention. The left calf is 3 cm greater in circumference than the right calf. He has pain with dorsiflexion, and there is an area of tenderness in the medial calf.

Which one of the following is the most appropriate next step in ruling out deep vein thrombosis in this patient?

A) D-dimer B) Ultrasonography C) Venography D) Impedance plethysmography

192. A 39-year-old male presents to the emergency department with a 2-hour history of chest discomfort, dyspnea, dizziness, and palpitations. He has no history of coronary artery disease. He states that he has had several similar episodes in the last year. On examination he has a temperature of 36.8°C (98.2°F), a respiratory rate of 25/min, a heart rate of 193 beats/min, a blood pressure of 134/82 mm Hg, and an O2 saturation of 96% on room air. The physical examination is otherwise normal. An EKG reveals a regular narrow QRS complex tachycardia with no visible P waves.

He converts to normal sinus rhythm with intravenous adenosine (Adenocard). Which one of the following would be most useful in the long-term management of this patient’s condition?

A) Adenosine B) Digoxin C) Vagal maneuvers D) Pacemaker placement E) Radiofrequency ablation

193. A 44-year-old male in the intensive-care unit develops acute respiratory distress syndrome (ARDS). Which one of the following has been shown to improve outcomes in this situation?

A) Surfactant B) Lower positive end-expiratory pressure (PEEP) settings C) Lower tidal volumes D) Aggressive fluid therapy E) Pulmonary artery catheters

56 194. A 49-year-old uninsured female with diabetes mellitus presents with painful burning of her feet, particularly at night. She has tried ibuprofen and acetaminophen without relief. Her last hemoglobin A1c was 7.1%. Her medications include metformin (Glucophage), glipizide (Glucotrol), lisinopril (Prinivil, Zestril), and lovastatin (Mevacor).

Which one of the following would be the best choice to treat her foot pain?

A) Amitriptyline B) Topiramate (Topamax) C) Fluoxetine (Prozac) D) Lamotrigine (Lamictal)

195. A 2-year-old male is brought in for an initial office visit. He just moved to your community to live with foster parents. On examination, you note a thin upper lip, a smooth philtrum, a flat nasal bridge, small palpebral fissures, a curved fifth finger (clinodactyly), and a widened upper palmar crease that ends between the second and third fingers.

These findings suggest which one of the following?

A) Trisomy 21 (Down syndrome) B) Marfan syndrome C) sequence (Potter syndrome) D) Fetal alcohol syndrome E) Prader-Willi syndrome

196. Which one of the following is most characteristic of the pain associated with acute pericarditis?

A) Improvement when sitting up and leaning forward B) Improvement when lying supine C) Worsening with the Valsalva maneuver D) Radiation to the right scapula E) Radiation to both arms

197. According to the American Diabetes Association, screening should be considered for which one of the following conditions in children with type 1 diabetes mellitus?

A) Hypothyroidism B) Cystic fibrosis C) Cushing syndrome D) Systemic lupus erythematosus E) Pancreatic pseudocysts

57 198. A 9-year-old male is brought to your office because he has developed a limp and refuses to bear weight on his right leg. On examination he has a temperature of 38.6°C (101.5°F) and pain with range of motion of the right hip. His WBC count and erythrocyte sedimentation rate are both elevated. A radiograph of the right hip is normal.

Which one of the following would be most appropriate at this point?

A) A repeat radiograph in 48 hours B) Ultrasonography of the right hip C) CT of the right hip D) MRI of the right hip E) A bone scan of the lumbar spine, right hip, and right femur

199. Prophylactic cholecystectomy for asymptomatic gallstones is indicated for patients with which one of the following?

A) Sickle cell disease B) A renal transplant C) Diabetes mellitus D) Cirrhosis

200. You have diagnosed chronic fatigue syndrome in a 32-year-old female. Her PHQ-9 is negative for depression. An evaluation for sleep disturbance and other comorbid disorders is also negative.

Which one of the following would be the most effective treatment?

A) Cognitive-behavioral therapy B) Interpersonal therapy C) Citalopram (Celexa) D) Methylphenidate (Ritalin)

201. For several years, a hypertensive 65-year-old female has been treated with hydrochlorothiazide, 25 mg/day; atenolol (Tenormin), 100 mg/day; and hydralazine, 50 mg 4 times/day. Her blood pressure has been well controlled on this regimen. Over the past 2 months she has experienced malaise, along with diffuse joint pains that involve symmetric sites in the fingers, hands, elbows, and knees. A pleural friction rub is noted on examination. Laboratory testing shows that the patient has mild anemia and leukopenia, with a negative rheumatoid factor and a positive antinuclear antibody (ANA) titer of 1:640.

Which one of the following would be the most appropriate INITIAL step?

A) Replace hydrochlorothiazide with furosemide (Lasix) B) Discontinue hydralazine C) Start prednisone, 40 mg/day orally D) Start hydroxychloroquine (Plaquenil), 400 mg/day E) Order renal function studies and anticipate that a renal biopsy will be needed

58 202. A 52-year-old mechanic complains of an irritation in his right eye lasting for 2 days. On direct visualization you see a small, dark foreign body on the periphery of the cornea and are able to remove it with no complications. However, there is a patch of reddish-brown discoloration extending several millimeters around the area where the foreign body had been.

Which one of the following is most appropriate for this patient?

A) Watchful waiting B) Irrigation with 0.9% saline solution under pressure C) An antibiotic ointment to be used every 2–4 hours D) Gentle debridement with a #11-blade scalpel E) Prompt ophthalmologic evaluation

203. A mother calls to ask your advice because her healthy 3-year-old, who has not been immunized against hepatitis A, attends day care with a child who was just diagnosed with the illness. You advise her that her child should receive

A) no prophylactic treatment B) hepatitis A vaccine only C) immunoglobulin only D) hepatitis A vaccine and immunoglobulin E) hepatitis A vaccine, along with other family members

204. A 74-year-old male with hypertension, diabetes mellitus, and stage 3 chronic renal insufficiency presents to your office with a request from a consulting ophthalmologist for risk assessment prior to cataract surgery. His medications include lisinopril (Prinivil, Zestril), 20 mg daily; amlodipine (Norvasc), 5 mg daily; aspirin, 81 mg daily; pravastatin (Pravachol), 20 mg daily; and glipizide (Glucotrol), 5 mg daily. His blood pressure is 126/72 mm Hg. His most recent laboratory tests from 2 months ago show a hemoglobin A1c of 7.2% (N 4.0–5.6) and a serum creatinine level of 1.8 mg/dL (N 0.6–1.3). He is in his normal state of health, and is able to walk 1–2 blocks before having to stop to rest.

Which one of the following would be most appropriate with regard to preoperative medical testing for this patient?

A) No preoperative medical testing B) A CBC C) An EKG and cardiac stress testing D) An EKG, and if results are abnormal, stress testing or echocardiography

205. In a patient without allergies who is admitted to the hospital for hip joint replacement, which one of the following is the recommended prophylactic antibiotic?

A) Ampicillin B) Ampicillin/sulbactam (Unasyn) C) Cefazolin D) Clindamycin (Cleocin) E) Vancomycin (Vancocin)

59 206. A 45-year-old female presents to your office with knee pain. She was playing volleyball yesterday when she collided with another player and was unable to continue playing because of pain in her knee. The knee was swollen this morning. She is able to walk but not without pain, and she also has pain when she attempts to bend her knee. On examination there is medial joint line tenderness and a positive Thessaly test.

Which one of the following is the most likely cause of her knee pain?

A) Osteoarthritis B) Anterior cruciate ligament tear C) Collateral ligament tear D) Medial meniscus tear E) Tibial plateau fracture

207. You suspect orthostatic hypotension in an elderly male who reports “dizziness” when standing up, and you decide to obtain recumbent and standing blood pressure measurements. After the patient rests in a supine position for 5 minutes, you measure his baseline blood pressure and then ask him to stand, which he does without a problem.

For how long should his blood pressure be periodically measured before considering the test complete?

A) 30 seconds B) 60 seconds C) 90 seconds D) 3 minutes E) 5 minutes

208. Which metabolic disturbance is most likely to cause the EKG findings shown below?

A) Hyperkalemia B) Hypercalcemia C) Alkalosis D) Hypothermia

209. A 30-year-old female asks about her contraceptive options. She has polycystic ovary syndrome and is currently being treated with spironolactone (Aldactone) for hirsutism.

Which one of the following should be used with caution, due to an increased risk of hyperkalemia?

A) Oral drospirenone/ethinyl estradiol (Yaz, Yasmin) B) Oral norethindrone/mestranol (Necon 1/50) C) Oral norgestimate/ethinyl estradiol (Ortho Tri-Cyclen) D) Transdermal norelgestromin/ethinyl estradiol (Ortho Evra) E) Depot medroxyprogesterone acetate (Depo-Provera)

60 210. The etiologic agent that causes erysipelas is

A) Staphylococcus aureus B) Haemophilus influenzae C) Streptococcus pyogenes D) Pseudomonas aeruginosa E) Rubivirus

211. A 45-year-old female is being treated for hypothyroidism with levothyroxine (Synthroid), 112 :g daily. She is still having persistent fatigue and weight gain despite her TSH value of 1.5 :U/mL (N 0.5–5.5).

In addition to evaluating this patient for other causes of her symptoms, which one of the following would be appropriate management of her thyroid medication at this time?

A) Continuing the current therapy B) Increasing the dosage C) Adding liothyronine (Cytomel) D) Switching to desiccated thyroid hormone (Armour Thyroid)

212. Which one of the following basal cell carcinomas is associated with the highest risk of recurrence?

A) A 7-mm lesion on the nose B) A 9-mm lesion on the forehead C) A 12-mm lesion on the shoulder D) A 17-mm lesion on the arm

213. A 52-year-old male presents for a routine physical examination. His laboratory results reveal an AST (SGOT) level of 124 U/L (N 10–40) and an ALT (SGPT) level of 36 U/L (N 10–55). His (-glutamyl transpeptidase (GGTP) level is also elevated.

The most likely cause of this abnormality is

A) hepatitis C B) hemochromatosis C) nonalcoholic fatty liver disease D) alcoholic liver disease E) statin-induced liver disease

214. Which one of the following is an appropriate treatment for tinea capitis?

A) Oral cephalosporins B) Oral griseofulvin C) Topical acyclovir (Zovirax) D) Topical ketoconazole (Nizoral) E) Topical miconazole (Monistat)

61 215. An 88-year-old male nursing-home patient is having problems with constant overflow incontinence. Intermittent catheterization has proven difficult due to urethral obstruction and his resistance to such procedures. He has dementia and generalized weakness as a result of multiple strokes and is bedbound, requiring total care for most activities of daily living. Examination shows a grade 3 coccygeal ulcer that has been present for several months, and a digital rectal examination demonstrates a large, irregular prostate.

Which one of the following is the best choice to quickly correct his incontinence?

A) Doxazosin (Cardura) B) Finasteride (Proscar) C) Tolterodine (Detrol) D) Long-term indwelling Foley catheter placement E) Referral for transurethral prostatectomy

216. A 45-year-old female presents to an urgent care center complaining of left-sided chest pain for the past 2 days. The pain is nonradiating and sharp in character, and increases with deep inspiration. She has no associated shortness of breath, cough, nausea, diaphoresis, or dizziness. She has no significant past medical history or recent travel history.

On examination she is afebrile, with a pulse rate of 102 beats/min, a blood pressure of 116/72 mm Hg, and a respiratory rate of 22/min. Her lungs are clear and her heartbeat is regular with no murmurs. Her lower extremities have no edema, tenderness, or varicosities.

Which one of the following is the most appropriate next step in her evaluation?

A) A high-sensitivity D-dimer test B) A troponin I level C) An antinuclear antibody level D) Ultrasound examination of the veins of the lower extremities E) Multidetector helical CT of the chest

217. Which one of the following organisms is the most common cause of cutaneous infections associated with intertrigo?

A) Candida albicans B) Pseudomonas aeruginosa C) Staphylococcus aureus D) Group A $-hemolytic Streptococcus E) Trichophyton mentagrophytes

62 218. A 4-year-old male presents with a 1-week history of a fever at or slightly above 38°C (101°F) that has responded poorly to antipyretics. The patient complains of photophobia, burning eyes, and a sore throat. On examination his eyes look red, his lips are red and cracked, and he has a “strawberry tongue.” In addition, his palms and soles are erythematous and there is periungual desquamation on his fingers and toes. He has minimally painful nodes located in the anterior cervical region, about 2×2 cm in size. A Streptococcus screen is negative.

The most appropriate management at this time would be

A) intramuscular benzathine penicillin G (Bicillin L-A), 600,000 U B) intravenous nafcillin C) aspirin and intravenous immune globulin D) prednisone, 2–3 mg/kg daily E) a fine-needle biopsy of the lymph nodes

219. A 48-year-old male presents with a 4-week history of rectal pain associated with minimal rectal bleeding. On examination there is a small tear of the anorectal mucosa at the 6 o’clock position.

The most appropriate initial treatment would be topical

A) botulinum toxin B) clobetasol (Temovate) C) capsaicin (Capzasin-HP, Zostrix) D) nitroglycerin

220. A 3-year-old male is brought to your office the day after he was stung by a honeybee. He developed a significant local reaction, with redness and swelling around the site of the sting on his forearm. He also had some swelling of his lips which lasted 2–3 hours. He was treated with oral diphenhydramine (Benadryl) at home and now his symptoms have completely resolved.

Which one of the following should be recommended for this patient?

A) An epinephrine autoinjector (EpiPen) B) Corticosteroids as needed for stings C) Immunotherapy for 1–2 years D) Reassurance only

63 221. A 54-year-old male sees you for a 6-month follow-up visit for hypertension. He feels well, but despite the fact that he takes his medications faithfully, his blood pressure averages 150/90 mm Hg. He has had an intensive workup for hypertension in the recent past, with normal repeat laboratory results, including a CBC, serum creatinine, an electrolyte panel, and a urinalysis. His medications include chlorthalidone, 12.5 mg daily; carvedilol (Coreg), 25 mg twice daily; amlodipine (Norvasc), 10 mg daily; and lisinopril (Prinivil, Zestril), 40 mg daily. He has been intolerant to clonidine (Catapres) in the past.

Which one of the following medication changes would be most reasonable?

A) Adding isosorbide mononitrate (Imdur) B) Adding spironolactone (Aldactone) C) Substituting furosemide (Lasix) for chlorthalidone D) Substituting losartan (Cozaar) for lisinopril

222. Which one of the following occurs with delirium tremens but is not usually seen with less severe forms of alcohol withdrawal?

A) Fever B) Hypertension C) Tachycardia D) Seizure E) Visual hallucinations

223. You attend the cesarean delivery of a full-term male infant with no prenatal risk factors other than breech position. The infant was received from the operating team crying and vigorous, with Apgar scores of 8 at 1 minute and 9 at 5 minutes.

An hour later, the infant becomes ashen-blue in color with an O2 saturation of 82%. He is alert and tachypneic, with a heart rate of 140 beats/min and a normal temperature. You order supplemental oxygen.

Which one of the following is the most appropriate next step in managing this patient?

A) Bag-mask ventilation with 100% oxygen B) Endotracheal intubation C) Prostaglandin E1 infusion D) An EKG E) A chest radiograph and laboratory studies

64 224. A 26-year-old African-American male presents with diffuse abdominal and back pain 5 days after being diagnosed with streptococcal pharyngitis. He appears mildly ill, with slight scleral icterus.

Laboratory Findings

Unconjugated bilirubin...... 3.2 mg/dL (N <0.2) Hemoglobin...... 9.6 g/dL (N 13.0–18.0) Hematocrit...... 29.4% (N 37.0–49.0) Peripheral blood smear...... keratocytes, blister cells, and polychromatic macrocytes

Which one of the following is the most likely diagnosis?

A) Thalassemia B) Sickle cell anemia C) Spherocytic hemolytic anemia D) Microangiopathic hemolytic anemia E) G6PD deficiency

225. A 62-year-old African-American male is admitted to the hospital for the third time in 6 months with heart failure. He has dyspnea with minimal activity. Echocardiography reveals an ejection fraction of 40%.

Which one of the following combinations of medications is most appropriate for long-term management of this patient?

A) Enalapril (Vasotec) plus digoxin B) Hydralazine plus isosorbide dinitrate C) Losartan (Cozaar) plus amlodipine (Norvasc) D) Spironolactone (Aldactone) plus bisoprolol (Zebeta)

226. A heroin overdose is most likely to cause acute

A) renal failure B) hepatic necrosis C) myocardial infarction D) pulmonary edema E) pelvic thrombophlebitis

65 227. A 56-year-old female comes to your office because she feels “swollen all over.” You do not notice any signs of edema, despite her claim that her rings no longer fit and that she has gone from a shoe size of 6 to a 7½ over the last 2 years. A review of systems reveals that she is sweating more than usual, feels fatigued, and often has a dull headache and diffuse arthralgias. She denies shortness of breath, chest pain, and abdominal symptoms. On examination she has no joint swelling, erythema, or tenderness. No skin abnormalities are noted.

This presentation is most consistent with which one of the following?

A) Acromegaly B) Cushing’s disease C) Polymyalgia rheumatica D) Scleroderma E) Systemic lupus erythematosus

228. A 35-year-old male has been admitted to the psychiatric inpatient unit for decompensation of schizophrenia. The staff reports that he drinks an estimated 10 liters of water per day. His serum sodium level is 128 mEq/L (N 135–145) and his serum osmolarity is 268 mOsm/kg (N 275–295).

Which one of the following findings would indicate that the etiology of this patient’s problem is psychogenic?

A) A urine sodium level of 60 mEq/L (N 15–250) B) A urine specific gravity of 1.020 (N 1.010–1.025) C) A urine osmolarity <100 mOsm/kg (N 500–800) D) Elevation of antidiuretic hormone E) Demonstrable pitting edema

229. A 67-year-old male with moderate to severe COPD has had several exacerbations in the past year, one requiring hospitalization. Regular use of which one of the following long-term treatments would be expected to reduce his risk for another exacerbation of COPD?

A) Oral theophylline $ B) Inhaled short-acting 2-agonists $ C) Inhaled long-acting 2-agonists D) Inhaled short-acting ipratropium bromide (Atrovent) E) Nasal oxygen

66 230. A 70-year-old female with a past history of hypertension and diabetes mellitus is hospitalized with pneumonia and treated with antibiotics. She subsequently develops two Clostridium difficile infections and is appropriately treated with antibiotics each time. Ten weeks after her initial hospitalization she has her third episode of C. difficile infection.

Which one of the following would be the most appropriate treatment?

A) A 14-day course of linezolid (Zyvox) B) A 14-day course of oral vancomycin (Vancocin) and metronidazole (Flagyl) C) A 14-day course of intravenous vancomycin D) A 4-week course of clindamycin (Cleocin) E) A 15-week oral vancomycin taper

231. An elderly male with mild dementia is involved in a motor vehicle accident, and his son is concerned that it may no longer be safe for him to drive. Which one of the following has the legal authority to revoke or restrict this patient’s driver’s license?

A) The patient’s son B) A psychiatric consultant C) The person designated as having power of attorney D) The family physician E) A representative of the state department of motor vehicles

232. A 12-year-old female is brought to the emergency department with an asthma exacerbation. Which one of the following indicates that her exacerbation may be life threatening?

A) A past need for systemic corticosteroids B) Inspiratory and expiratory wheezing in both lung fields C) Paradoxical chest movement D) A PaCO2 <35 mm Hg E) An FEV1 that is 60% of expected after initial treatment in the emergency department

233. A 6-year-old female is brought to your office because of a pruritic rash on her lower abdomen that has been present for about 6 months. Her mother says that at times the rash will completely clear up only to reappear at the same location. The child has had no other skin disorders and is otherwise healthy. The rash is shown below.

Which one of the following is the most likely cause of this condition?

A) Eczema B) Herpes zoster C) Nickel sensitivity D) Squamous cell carcinoma E) Tinea corporis

67 234. An 18-year-old high-school football player comes to the walk-in clinic the morning after a game in which he injured the middle finger of his right hand. During the game he grabbed the jersey of an opposing player as he attempted to tackle him. The opposing player pulled away from his grasp and the patient immediately felt pain in the distal interphalangeal (DIP) joint of the affected finger. When you examine him he cannot flex the affected finger at the DIP joint. Radiographs show a bony fragment at the volar surface of the proximal distal phalanx.

Which one of the following would be the most appropriate management?

A) Referral to a hand surgeon B) Splinting the DIP joint in flexion for 4–6 weeks C) Splinting the DIP joint in extension for 4–6 weeks D) Unrestricted range of motion exercises E) Buddy taping the affected finger to the adjacent finger

235. A 3-year-old male went on a camping trip and stayed in the same tent as his cousin for 4 nights. On the first day of the trip the cousin developed a bad cough. They returned from the trip yesterday, and the cousin has just been diagnosed with laboratory-confirmed pertussis. The patient is up to date on all immunizations and had his last DTaP over 1 year ago. He has a 4-month-old sister at home.

Which one of the following is the most appropriate recommendation for this patient?

A) A DTaP vaccine booster B) Oral azithromycin (Zithromax) as a single dose C) Oral azithromycin for 5 days D) Tetracycline for 5 days E) No prophylaxis

236. Which one of the following is associated with vitamin B12 deficiency?

A) Prednisone B) Metformin (Glucophage) C) Insulin glargine (Lantus) D) Gabapentin (Neurontin) E) Risperidone (Risperdal)

68 237. A 27-year-old female reports a year-long history of amenorrhea. She has also had an increased amount of milky discharge from her nipples over the past several months and has lost all interest in sex for the past 6 months. She denies any drug or medication use other than occasional over-the-counter analgesics for frequent headaches. A thorough physical examination confirms the presence of an easily expressed milky discharge, as well as vaginal dryness. A pregnancy test is negative.

Which one of the following tests would be most appropriate at this point?

A) A serum prolactin level B) Ultrasonography of the breasts C) MRI of the sella turcica D) A dexamethasone suppression test

238. A 24-year-old female presents to your office with a complaint of fatigue and concentration problems. She thinks she is depressed because she is always worried about something. She says she has difficulty sleeping at night because she can’t stop thinking and worrying. Her constant worrying has started to affect her relationships and she is finding it difficult to work. She is willing to go to therapy but would also like to start a medication.

Which one of the following would be most appropriate?

A) Buspirone (BuSpar) B) Lorazepam (Ativan) C) Clonazepam (Klonopin) D) Sertraline (Zoloft) E) Pregabalin (Lyrica)

239. A 54-year-old white female presents to your office for a well care visit. She has no physical complaints and her last examination was 5 years ago. Routine laboratory work reveals an alkaline phosphatase level of 300 U/L (N <135). Ultrasonography of the liver is normal. You are concerned about the possibility of primary biliary cirrhosis.

Which one of the following would be most appropriate for further evaluation of this patient?

A) An antinuclear antibody level B) An anti–smooth muscle antibody level C) An antimitochondrial antibody level D) Magnetic resonance cholangiopancreatography (MRCP) E) A liver biopsy

240. Over-the-counter analgesic/decongestant combinations have been proven harmful for individuals in which age range?

A) 6 months–2 years B) 2 years–6 years C) 18 years–50 years D) >75 years

69